Download as pdf
Download as pdf
You are on page 1of 513
Preface 1. Dime Intoatuetion Physical Quantity Types of Physieal Quantity Fundamental and Derived Quantities al ang Derived Units 1 Application Exercise Lol incusions ofa Physical Quantity Methods to Find Dimensions of Physical tant or Coelicients Applications of Dimensional Analyst Limitations of Dimensional Analysis Application Exercise 1.2 Conee Significant Figures Roundit or ‘ant Figures in Calculation Order of Magnitude Concept Application Exercise 13 Errors of Measurement Propagation of Errors Concept Application Exercise 14 ‘Vernier Calipers Reading a Vernier Callipers Zero Error and Zero Correction Positive and Negative Zero Error Calculating Positive Zero Error Calculating Negative Zero Error Screw Gauge (or Micrometer Screw) Constants of the Serew Gauge Zero Error and Zero Correction Solved Examples Exercises Single Correct Answer Type Multiple Correct Answers Type Linked Comprehension Type _______,___—y— 1 tae Ww ul ul ul 12 12 13 13 13 14 La 18 1s 19 19 1.10 110 Lu LAL Lu 1S Las Las 1.16 1.16 17 Ly LIT 117 1.18 119 1.23 1.23 127 1.28 2 Matrix Match Type ‘Numerical Value Type ‘i ° Archives Ib Answers Key 135 snasic Mathematics a Introduction Fi Elementary Algebra 20 ‘Common Formulae 4 polynomial, Linear, and Quadratic Equations dW Binomial Expression and ‘Theorem 2 Concept Application Exercise 2.1 32 Elementary ‘Trigonometry ' 22 ‘Angle and Systems of Measurement of an Angle — 2.2 Four Quadrants and Sign Conventions 23 Limits of the ‘Values of Trigonometric Ratios 23 Graphs of Sine and Cosine Functions 24 ‘Some Important Trigonometric Formulae 14 “rigonometscal Ratios of General Angles (Reduction Formulae) 24 Inverse Trigonometric Functions 25 Sine and Cosine Rute for Triangles 25 Concept Application Exercise 2.2 25 Basic Coordinate Geometry 26 Origin 26 Axis 26 Position of a Point 2.6 Distance Formulae 27 Slope of a Line 27 Straight Line Equations 27 Dependent and Independent Variables 28 Plotting the Velocity-Time Relation of Particle Moving with Constant Acceleration 29 Concept Application Exercise 2.3 29 Parabola: Quadratic Equations 2.10 Concept Application Exercise 2.4 212 Function 2.13 Differentiation 213 213 Limit of a Function B® Ww Contents ee Different Coefficient or Derivative ofa Function 2.14 ee eae oa) Comat By Rimphical Representation of Derivative ota Am y Petites Wiccan ing Componsof¢ e aaen afte ‘Acceleration emeera oer Finding a eso if and y Comp . Notation 21S ‘setor are Given of a Vector . Properties of Deatne zie aes mea Conroe ny pantaiv® ofa Power Function ae ‘dion of Ventas by Means of Components 34 Derivative of Function sn 217 Direction Cosines ay Derivative of Function coe ¢ 27 ce 3 Differentiation of Commonly Used Functions 2.18 peor Ag Product Rule 218 eat Exercise 3.3 3m Quotient Rule 219 ee 38 Chain Rule or “Outside Inside” Rule 2.19 Pr eee 3.18 Double Differentiation 221 Dot Produ of Unt Veron 319 Applications of Derivative in Physics 221 Vector Product (Cross Product) 31 Maximum and Minimum Values of a Funetion 2.22 Ce mplication Exercise 3.4 3.24 Concept Application Exercise 2.5 223 peas 34 Elementary Integration 2.24 Solved Examples 3.28 Integration as Inverse of Differentiation 2.25 eee 3.28 Finding Integration of Constant Function 225 eee opie : 3.31 Some Important Properties of Integration 225 meee oo 3.33 ‘Standard Formulae for Integration 225 ae Tipe 3.34 Definite Integral of a Function 2.26 ‘Mumerical Value Type 3.35 Algebraic Method to Evaluate Definite Integral 2.27 Archives 3.36 Properties of Definite Integral 227 tes 3.37 Rule of Substitution 2.27 “ Geometrical Significance of a Definite Integral 2.28 Kinematics I ieee Geometrical Method to Evaluate Definite Integral 2.28 Introduction ‘ ' Concept Application Exercise 2.6 2.29 Translatory Motion 7 Applications of Integration in One-Dimensional Frame of Reference 4 Motion 2.29 Rest and Motion 41 Derivations of Equations of Motions by Calculus Trajectory 41 Method 2.29 Trajectory Equation 42 Graphical Interpretation of Integration 2.30 Finding Trajectory Equation 42 Concept Application Exercise 2.7 231 Displacement and Distance 42 Solved Examples 232 Expressing Displacement in Case of Motion in 3. Vectors 3-41.38 (One Dissension vi Vectors and Scalars 31 eee “4 Representation of Vectors 3 7 Addition of Vectors 32 ee Ela a Triangle Law of Vector Addition 32 Parallelogram Law of Vector Addition 32 emma . _ eee 3 Average Acceleration 47 Polygon Law of Vector Addition 33 raat! Avocleat " is Some Properties of Vector Addition 33 ee Application Exerise 4.1 cau) Concept Application Exercise 3.] 34 amas ae au Vector Addition by Analytical Method 35 7 ao yor Uniformly Accelerated Motion Concept Application Exercise 3.2 38 Displacer Late) ee o a Unit Vectors 38 ‘Placement of a Particle in n® Second of its Motion in Uniformly Accelerated Motion 4.12 _ - ~~ projectile Motion on an Incined Pang ———2tes_v mn Eee TG aid Concept Application Exercise $.4 524 Onn Evercise 4.2 ° an Gare imeesional Motion ina Vertical Line ans Relative iain * . $35 Nretion Under Gravity) nA Relative Velocity in TWo Dimensions 527 Caen Pen an Incined Plane a “Analysis of Relative Motion in Genera 527 ie Aer ‘ation Exercise 4.3 a a Graphical Method to Find Relative Velo. $27 etn Alone Other Sloe Lines Relative Motion in River Flow YY sa8 Tee ttonin One Dimension “ Bain-Man Problems 53 River-Miog gr PreaehSeparaton : fe wind-Airplane Problems 534 Concept sve in One Dimension : 7 Shortest Distance Between Two Moving Particles o Graphs in Motion in Onc bre 428 Velocity of Approach * re mn in One Dimension slication Exercise 5. Sal lotion Diag > Concept Application Exercise 5.5 wa Graphical Re ‘Examples as “SU eresetaton for Motion 428 ae a au '© Analyze and Draw th 428 vere 532 Position-Time Relation as 429 ‘single Correct Answer Type 552 Velocity-Time Graph agt Multiple Correct Answers Type es Acceler ~ in yension Ty Motions cen grime Graph of Various Types of Linked Comprehension Type 560 Vea 8 Particle 434 Matrix Match Type 58 — fs isplacement Graph 4.40 ‘Numerical Value Type 5.65 Acceleration 1 — —- Displacement Graph 4a Archives 567 ‘cation Exercise 4.5 442 inswers Ke 5. Solved Examples ia Answes ” Exercises 455 | Newton's Laws of Motion (Without Friction) 6.1-5.76 Single Correct Answer Type 455 Introduction 61 Multiple Correct Answers Type 461 ‘Newtonian Mechanics 61 Linked Comprehension Type 4.64 Concept of Force 61 Matrix Match Type 466 Classification of Forces 61 Numerical Value Type 468 Newton's Laws of Motion 61 Archives 470 Newton's First Law of Motion 61 Answers Key 47 Inertial Frame of Reference 62 ‘5, Kinematics II 51-568 ‘Newton's Second Law of Motion 62 Principle of Conservation of Linear Momentum 63 Introduction 51 Impulse 63 Velocity and Acceleration in Two-Dimensional Motion 5.1 Principles of Physical Independence of Motions 5.2 Concept Application Exercise 6.1 63 ‘Two-Dimensional Motion with Constant Newton's Third Law of motion 66 Acceleration 53 Free-Body Diagrams 6 Concept Application Exercise 5.1 54 Weight 67 Projectile Motion 54 Normal Force 67 Principles of Physical Independence of Motions Representing Normal Reaction in Different in Projectile Cases 34 Situations 67 Calculation of Various Parameters in Projectile Representing Normal Reactions and Weight Motion 5.6 in Free-Body Diagrams 68 Concept Application Exercise 52 $13 Tension 68 Horizontal Projectile S14 Ph resentation of Tension Force in Different Situations 69 ght at Certain Angle with au Projectile ha Height at Ce sc fia Friction ‘o a = rom a Moving Frame 5.16 Equilibrium ofa Particle 69 ne ‘im Velocity and Angle to Hit a Given Point 5.22 ‘Concurrent Forces 69 ae Collision of a Projectile with @ Wall an : ie Theorem 69 sti ms a ercise $3 523 roblem-Solving by Applying Newton's Laws 6.10 jon Ex Concept Application Concept Application Exercise 6.2 6.13 Contents Dynamics of Particles: Translational Motion of Accelerated Bodies 6.14 Analysis of Newton’s Laws of Motion in Connected Bodies: Problems Based on Normal Reaction Apparent Weight Concept Application Exercise 6.3 Problems Based on Blocks Connected with Strings Problems of String with Mass Concept Application Exercise 6.4 Non-inertial Frame of Reference ‘and Pseudo (Fictitious) Force Concept Application Exercise 6.5 Constraint Relation Pulley Constraint | ‘Wedge Constraint: Normal Constraint Pulley and Wedge Constraint General Constraints Concept Application Exercise 6.6 Spring Force and Combinations of Springs Force Constant of Composite Springs Springs in Parallel Springs in Series Concept Application Exercise 6.7 Solved Examples Exercises Single Correct Answer Type Multiple Correct Answers Type Linked Comprehension Tipe Matrix Match Type Numerical Value Type Archives Answers Key 7. Newton’s Laws of Motion (With Friction) Introduction Friction and Frictional Force Static and Kinetic Friction Properties of Friction Direction of Frictional Force Concept Application Exercise 7.1 Angle of Friction Pull is Easier Than Push Minimum Force Required to Move a Block Angle of Repose or Angle of Sliding Two Blocks in Contact Moving on an Inclined Plane Concept Application Fvercise 7.2 Analysis of Frictional Force Between Two Blocks in Contact: Condition of Existence of Static Friction Friction Acting on Multiple Surfaces 616 617 618 619 622 623 624 628 629 629 635 636 639 6.40 642 642 642 642 645 647 6.58 658 6.66 668 71 673 675 6.76 71-556 a 71 7 2 72 79 7.10 7.10 7.10 7 7.16 707 718 721 Jn Lower Block If Foree Applied or 7a; Ir Force Applied on Upper Block svnalysing Frietional Force While Walking 7.35 Concept Application Exercise 73 Solved Examples 14 Exercises 737 ‘Single Correct Answer Type 737 Multiple Correct Answers Type 748 Linked Comprehension Type 246 Matrix Match Type 730 Numerical Value Type 12 54 ‘rower Ke 7 8. Work, Energy and Power A AB6T Introduction es ‘Work Done by a Force Nature of Work Done 82 Positive Work (0° << 90°) 82 Negative Work (90° < 0 180°) 82 Zero Work 82 ‘Work Depends on the Frame of Ref 82 Work Done by a Variable Force 83 ‘Graphical Interpretation of Work Done 84 Work Done by Different Forces 85 Work Done by Gravity 85 Work Done by a Pair of Interacting Forees 85 Work Done by Statie Friction 86 Work Done by Kinetic Friction 88 Work Done by Spring Force 8.10 Work Done by a Pseudo (Inertial) Force Bi Concept Application Exercise 8.1 sir Kinetic Energy an Work-Energy Theorem 812 Concept Application Exercise 8.2 818 Conservative and Non-Conservative Forces 8.19 Conservative Force 8.19 Non-Conservative Force 8.19 Potential Energy 820 Gravitational Potential Energy 821 Elastic Potential Energy Stored in a Spring 8.22 Change in Potential Energy 322 | Three-Dimensional Formula for Potential Energy 8.23 | Potential Energy Curve 323 Energy Diagram for a Typical Attractive | Two-Atom System 8.24 | Nature of Force 8.25) Stability 225 Concept Application Exercise 8.3 827 Mechanical Energy and its Conservation 8.28 eee Tuming of @ Vehicle on Horizon 7 Content Meche Pltion Beenie 4 3 Banking of Roads Creer Road a " Pa cen ower = Centrifugal Force ce Concept Appicanan wi Pam i a Concept Application Exercise 9.3 921 Sonn EP tton Bxerise 83 We Motion i verti ctle a ‘eorises Pes 8 " Condition of Completing Vertical ij 924 oe 8 a Condition for Osan: The Speed Became, 225 Matigte oe Answer Type bat Before Tension 1S Zero Linke sot Answers Tipe 353 Condition for Leaving the Circle 925 Matrix mebrehension Bpe 857 Tension Becomes Zero Before Speed Man Meth Tp 3.60. Condition fr Looping te Loop in Some Oey 225 Archives te Tipe ea Css 9. Answers Key 8.65 Motion ofa Particle on a Spherical Surface a © $67 some Special Cases of Circular Motion oa 9. Circular Motion 9.1-9.60 Circular Motion in Non-netal Frame of Reference e Introduction “a Circular Motion on an Inclined Plane oan Kinematics of Circular Mee ol Concept Application Exercise 9.4 ia a and Non-Uniform Circular Motion 91 Solved ae 9.34 FE norant Terminology Related to Circular Motion 9 Breit nan 943 ‘pressing Angular Displacement in Vector Form 9.1 Single Correct Answer Type 943 Relation Between Linear and Angular Quantities 9.2 ‘Multiple Correct Answers Type 948 Frequency and Time Period 92 Linked Comprehension Type A Instantaneous Angular Acceleration 93 ‘Matrix Match Tipe a Motion with Constant Angular Acceleration 93 Numerical Value Type 9.56 Relation Between Linear and. Angular Velocity 94 Archives 9.58 Relative Angular Velocity 94 Answers Key 980 Acceleration in Circular Motion 97 Tangential and Radial Acceleration 97 ie S1-S.i67 Total Acceleration 9g Chapter | Si Finding Centripetal Acceleration og Chapter? SAS Acceleration in Circular Motion: Vector Approach 9.9 Chapter 3 8.9 Radius of Curvature 9.19 Chapter 4 830 Concept Application Exercise 9.1 gy Chapter 5 sag Dynamics of Circular Motion 9.12 Chapter 6 sm Centripetal Force 9.12 Chapter 7 $93 Applications of Newton's Laws of Motion Chapter 8 S116 in Circular Motion 9.12 Chapter 9 S.144 Non-Uniform Circular Motion 915A ends comers? at JEE Main Questions (All Sets) Ad-AA UNTRODUCTION Like all other . physies is. based on experimental observations and quantitative measurements. The main objectives, of physics are to identify a limited mumber of fundamental laws that govern natural phenomena and use them to develop theories that can predict the results of future experiments. The fundamental laws used in developing theories are expressed in the language of mathematics, the tool that provides a bridge between theory and experiment. PHYSICAL QUANTITY A quantity which can be measured and by which various physical happenings can be explained and expressed in the form of laws is called a physical quantity. For example, length, mass, time, force, ete cienct Measurement is necessary to determine the magnitude of a physical quantity, to compare two similar physical quantities, sand to prove phy’sical laws or equations, A physical quantity is represented completely by its magnitude and unit. For example, 10 m means a length which is 10 times, the unit of length. Here 10 represents the numerical value of the given quantity and meter represents the unit of quantity under consideration. Thus, in expressing a physical quantity, we choose ‘unit and then find that how many times that unit is contained in the given physical quantity, Le Physical quantity (Q) = Magnitude x Unit =n xu where n represents the numerical value and w represents the unit. Thus, while expressing a definite amount of physical quan- tiny, itis clear that as the unit w changes, the magnitude (n) will also change but product nu will remain same ie, mu= constant ; ‘nu, = constant = = ie, the magnitude of physical quantity and units are inversely proportional to each other. Larger the unit, smaller will be the ‘magnitude AYES OF PHYSICAL QUANTITY Ratio (numerical value only) When a physical quantity 1s a ratio of two similar quantities, it has no unit. For example, Relative density = Density of object/Density of water at 4°C Refractive index = Velocity of light in air/Velocity of light in ‘medium Strain = Change in dimension/Original dimension ‘Angle is an exceptional physical quantity, which though is a ratio of two similar physical quantities (angle = arc/radius) but slill requires a unit (degrees or radians) to specify it along with its numerical value. Scalar (magnitude only): These quantities do not have any direction, e.g, length, time, work, energy, ete “The magnitude of a physical quantity can be negative. In that cease, negative sign indicates that the numerical value of the quan- tity under consideration is negative. It does not specify direction. ‘Scalar quantities can be added or subtracted with the help of following ordinary laws of addition or subtraction. Vector (magnitude and direction) Example of vectors are displacement, velocity, acceleration, force, etc. ‘Vector physical quantities can be added or subtracted accord- ing to the vector laws of addition. These laws are different from the laws of ordinary addition, FUNDAMENTAL AND DERIVED QUANTITIES Fundamental quantities Out of a large number of physical ‘quantities which exist in nature, there are only few quantities which are independent of al other quantities and do notrequire the help of any other physical quantity for their definition. Therefore, these are called absolute quantities, These quantities are also called fundamental or base quantities, as all other quantities are based ‘upon and can be expressed in terms of these quantities. Derived quantities All other physical quantities can be derived by suitable multiplication or division of different powers of fundamental quantities. These are, therefore, called derived quantities. For example, if length is defined as a fundamental quantity, then area and volume are derived from length and are expressed in term of length with power 2 and 3 over the term of length, Table: Fundamental quantities in SI system and their units [ie ntenay | oo 1. Mass Kilogram kg (2. Leng Meter m 3. Time Second | s 4. Temperature |Kewin K 5. Luminous testy ‘conden | Od 6, |Blectric Current | Aropere A 7. Amount of Substance | Mole mmol Supplementary quantities in SI system and their units Pantity. 7) Name oto 1.2 Mechanics 1 _ FUNDAMENTAL AND DERIVED UNITS ‘Normally each physical quantity requires a unit or standard for its specification so it appears that there must be as many units as there ‘are physical quantities, However, itis not so. thas been found that if in mechanics we arbitrarily choose units of any three physical “quantities we can express the units of all other physical quantities in mechanics in terms of these. Arbitrarily, the physical quantities ‘mass, length, and rime are choosen for this purpose. So any unit ‘of mass, length, and time in mechanics is called a fundamental, absolute, or base unit. Other units which can be expressed in terms of fundamental units are called derived units. For example, light year or km is fundamental units as itis a unit of length while s, rm or kgm! are derived units as these are derived from units of time, mass, and length, respectively System of units A complete set of units, both fundamental and derived, for all kinds of physical quantities is called system of Table: Prefixes used for iferent powers of 10 units, The common systems are given in the table below: | system and their units Table: Fundamental quantities in SI four more “fundamental units (in addition to three basic units) quantities in electricity, magnetism, heat, and light. ASI PREFIXES: ‘The magnitudes of physical quantities vary over a wide range. ‘The standard prefixes for magnitude too large or too small to be expressed more compactly for certain powers of 1Oare mentioned in the table below. Given that F = 5 N. Convert it into CGS system. e kgxm (10g) (i = (5) (0B) 000m) s : Fas =5 10° in CGS system) Tews (#2) abo caetdyae aS particle is moving with velocity v = 90 ‘velocity ofthe particle into mis. aoe ‘To convert Minto ™., multiply by >. Tour” geo PY ag Convert 78 pm into jm. HD Let 7pm=()um or 7x 108) m=. 7 We get x=7 x 10% ‘ needs toem So, 7 pm = (7 * 10°) um. ‘A new unit of length is chos te wile mae i senate aos ace of ‘new unit if i cover this dunce, oY UAE Tigh takes 8 min and 20 so IEE Here, speed of light in va ; acuum, ¢= it Time taken by light of sun to aes es . mee min and 20 seconds = (8 x 60 +20). = ‘Therefore, distance between the Sun and E an S= 1 new unit i “ it of length s-! x = 500 new units of engin n° J State True or false: (@ Radian is the uni supplementary qin n mit. ) ae Unit is always written in si form, e.g a ingular form, e. © Ifa unitis na ie unit is im a ted afer 4 person, the unit is not Mt of plane angle and it is a 7 | initial tote, | @ A+ BH @ IA + B= C~D, then units of 4, B, Cand D swat be same. Fill in the blanks: (a) The unit of work ® te unit of power is .., (©) The unit 7 © ae nitof energy Per unit volume is... (a) () and energy are ... The volume of a cube of side 1 cm is equal to....m’. ‘The surface area ofa solid i solid cylinder of radius 2.0 em and height 10 cm is equal to ...(mm). Avehicle moving with a speed of 18 km h! coves min Is. +(@)_ The relative density of lead is 11.3 Its density is... gem or kgm? 4. Fill in the blanks by suitable conversion of units (a) Lkgms?=.... gem’s? (b) 3.0ms?=...kmb? (© G=6.67 « 10" Nm (kg)? ee ere ‘ © (cmy's* aTme Tue == @) Tue (@) Toe 2. (a) kg m*s* (b) W (o) kg mr 3.(a)10* = (B)1.5*10' (Sm @ 113 108g a 4. (a) 10” (b) 3.9 * 10° (c) 6.67 ~ 10% - ‘ DIMENSIONS OF A ICAL QUANTITY When a derived quantity is expressed in terms of fundamental quantities, it is written as a product of different powers of the fundamental quantities. The powers to which fundamental ‘quantities must be raised in order to express the given physical quantity are called its dimensions. “To make it more clear, consider the physical quantity force: Force = Mass x Acceleration Mass Velocity _ Mass x Length/time Time me © = Mass x Length * (Time)? Thus, the dimensions aeorce are | in mass, 1 in length and -2.in time. Here the physical quantity that is expressed in terms oF he base quantities is enclosed in square brackets 10 indicate t a 1e equation is among dimensions and not among magnitudes: Thus, (can be written as (force] = (MLT") = hysical quantity in terms of the a a ated the dmesonl eyulion. we only the RE i termed Sof the equation, the expression is diene at ome Tes, the dimensional formula for force is (MLT™), Dimenslons and Measurement 1.3, JMETHODS TO FIND DIMENSIONS OF PHYSICAL CONSTANT OR COEFFICIENTS As ns of a physical quantity are unique, we write any formula or equation incorporating the given constant and then by substituting the dimensional formulae of all other quantities, we ccan find the dimensions of the required constant or coefficient. + Hight, width, radius, displacement, etc, are a kindof length. ‘So we ean say that their dimension is (L]. Here [Height] can ‘be read as “dimension of height.” Area of square = Length x Width [Area] = [Length] x [Width] = (Ex (= (27) Area of circle = 2 [Area] = [2]bI- (WI = 2) Here ris not a kind of length or mass or time, so x should not effect the dimension of area ‘+ Volume of cube = [Area] x (Height) [Volume] = (] x (Z] = (L"] ae) Volume of sphere = 57 ie) [Volume] = if alr -ow So dimension of volume will be always [L?] whether itis ‘volume of a cuboid or volume of sphere. Mass = Daw = Volume UMass) 1M yap (Density) = Famed (ay) + Veto = Displacement Time = Displacement] _ (2) _pyy0p17-1 “ [Time] (7) wer?) Change in velocity. Time Acceleration = ta- 4% ur) OM IT Linear momentum (P) = Mass x Velocity (P}= (MUM) = (ML = (wes) Mass x Acceleration UF] = Ma = (MILT?) = [ML'T] Work or energy = Foree x Displacement ery [Work] = [Force]{Displacement] = [MILT YL] = (WLeT?] + Powor= Moe Time a (Work) (MATA) _igeip2ry (Power) Time] ics T oe - 14 Mech «Pressure = FORE sure = TS o (Force) MTD ygty-tr [Pressure] = {Areal iz} DIMENSIONS OF ANGULAR QUANTITIES Radius + Angular displacement = — LEV _5481"7"} (Dimensionless) (0 FRadius) (2) aoe gular dispa + Angular velesty (@)= w= FE eae a Charge in angular velocity Angular acceleration = CUES 2 ARES _ [do] _ WO LT _yaye7 tanta a ‘Torque = Forve x Lever arm [Torque] = [Force] x [Lever arm] ML'T]x(L]= [MET] Gravitational constant: According to Newton's law of gravitation, IMI APPLICATI A ANALYSIS To find the unit of a physical quantity in a given system of units By writing the definition or formula for the physical quantity, we find its dimensions. Now in the dimensional formula replacing M, L and T by the fundamental units of the required system, we get the unit of physical quantity. However, sometimes to this unit, we further assign a specific name, ¢g,, Work = Force x Displacement So (W] = [MLT*] x [L]=[ML'T*] So, its units in CGS system will be g cms which is called erg while in MKS system will be kg m? s* which is called joule, To convert a physical quantity from one system to the other ‘The measure of a physical quantity is nu = constant. Ifa physical quantity X has dimensional formula [M*L!T and if the derived units of that physical quantity in two systems are [MpLbT;] and (M3247; ] and n, and n, be the numerical values in the two systems, respectively, then m[up] mUMPLTE = mI MELaTS) mu, = “reer Te My fe =a omen [4] [2 4 ‘and 7, are the funda fe first (known) system an mental units of mass, length, d M,, Lay and T, are af mass, length, and time in the second tly. Thus, knowing the values of the numerical value in one may be evaluated. where My bys and time in th the fundamental unit (unknown) system, fundamental units in system, the mu respectiv ‘two systems and merical value in the other system iy unit of force and has dimensional HED Newion is the SI formula (M'LIT?}, tea = 140 IN=1kgms? =le= So — m CGS system _M,=1ke M=18 a] L=im L=lom a T, tis] Is. 7 , = 1[10%s] [totem | is lg lem Is. TSBBD Joule: SI system; erg: CGS system Work = Force x Distance = Mass x Acceleration x Length = Mass x Length Dimensions of work = (W] = [M'L°T?] 2. a=1,b=2,c=-2. Now - Styystem | CGS system er 7 Here N, 7 +. Using N,= Ny [ = [as Ig Value of G in CGS unit = 6.67 x 10* cm’ gs? sso (a ee cso ET some 2 G =6.67 x 10"" MKS units system, the magnitude of the force is 100 dynes. In system where the fundamental physical quantities are ‘meter, and minute, find the magnitude of the force, HERB Wehaven, = 100,M,=1g,L,=10m,7,=1sandM,=1kg, 1,=1m,T,=1 min, Thedimensional formulaofforcesis(M'L'T?], SITET ona fe we have oe Dimensions and Measurenent = ‘We know that the dimension of Young’s modulus is [MT], ic.,a=1,b==1 CGS unit: g env" s? and MKS unit: kg mvs * By using the conversion formula: +. Conversion factor [atl wth een ht 42 3 age Urge ‘Suppose we employ a system of units ‘in which the unit of mass equals cckg, the unit of length equals Pai a tere arta rae 42 a'B*7? in terms of the new units. dimensional formula of energy is (ML: Comparing with [M*L'T}, we find that a = 1, b= 2, ¢=-2. baled ~ofesl les] ls] “eer Now, 1, ° try oof Teele tg | [tm | [1s = 54% 1 x 1000 x [3600] Hence, 54 kmh! = 15 ms". $4x1000 _ 3600 icone a? 4.6 Mechanics To check the dimensional correctness of a given physical relation This is based on the principle of homogeneity. According to this principle, the dimensions of each term on both sides of an equation must be the same. IfX = 44 (BC) + VDEF, then according to the principle of homogeneity, we have [x] = [4] = [(BCY] = [VDEF}, If the dimensions of each term on both sides are same, the equation is dimensionally correct, otherwise not. A dimensionally ‘correct equation may or may not be physically correct. Let us check the dimensional correctness of the relation, veutat. (SGED Here u represents the initial velocity, v the final velocity, a the uniform acceleration, and f the time. The dimensional formula of w is (M'LT"}. The dimensional formula of v is [M°LT™]. The dimensional formula of at is (M*LT2]{7] = [MPLT™}. Here the dimensions of every term in the given physical relation are the same, hence the given physical relation is dimensionally correct. Check the accuracy of relation v* — u? = 2as, where v and u are final and initial velocities, a is the acceleration, and s is the distance, EBRBD We have + — 1? = 2as. Checking the dimensions on both sides, we get LHS = [LTP - (LP = (1279) - [74] = [22 7] RHS =(L! 7) (J]= (22 F] Comparing LHS and RS, we find LHS = RHS. Hence, the formula is dimensionally correct. Check whether te relation S=u +a? is dimensionally correct or not, where symbols have their usual meaning. 1 TBM We have S= ut 3a Checking the dimensions on both sides, LHS = [M°L'T] (£7) [7] + (LT? [7"] = [MOLD] + (MOLT) =(MLT) Comparing the LHS and RHS, we get LHS = RHS Hence the formula is dimensionally correct. Find out the unit and dimensions of the constants a and b in i van der Waal’s equation (e+ (V-b)=RT, where p pressure, vis volume, Ris gas constant, and 7's temperature, [BRED We can add and subtract only like quantities. a => Dimensions of P= Dimensions of 7 @ and dimensions of v= Dimensions of b (i) From (i), Dimensions of a = Dimensions of P x Dimensions of 1? [a] = [ML 13] x [DY = (ML T] N nit of p x Unit of V0 = — x m® = Nm* Unit ofa From (i), (0)= (Y= WeLT] So unit of b = Unit of V= m* ‘A famous relation in physics relates the moving mass m to the rest mass m, of a particle in terms of its speed v and the speed of light c. (This relation first arose as a consequence of the special theory of relativity due to Albert Einstein). A boy recalls the relation almost correctly but forgets where to put a 5 Sie - Guess where op the constant c. He writes m the missing c. (BBE According to the principle of homogeneity of dimen- sions, powers of M, L, T on either side of the formula must be equal. For this, on RHS, the denominator (1 — +)! should be dimensionless. Therefore, instead of (1 - v7)", we should write (vie). Hence, the correct formula would be Mp ie Bs a research tool to derive new relations If one knows the dependency of a physical quantity on other quantities and if the dependency is of the product type, then using the method of dimensional analysis, a relation between the quantities can be derived. Let us understand this point through following examples. Time period of a simple pendulum Let the time period of a simple Pendulum be a function of the mass of the bob (m), effective length (J), and acceleration due to gravity (g), then assuming the function to be the product of power function of m, 1, and g, ie, 7= Km Pg’, where K = dimensionless constant. If the above relation is dimensionally correct, then by substituting the dimensions of quantities, (7) = (My (Ly (LT2}° or (MLD) = LT, Equating the exponents of similar quantities, we get x = 0 y= 12, end 2=-1/2, vee So, the required physical relation becomes T = K lig. The value of dimensionless constant is found (2 ) through experiments, 0 T'= 2x flig Stoke’s law: When a small sphere moves at low speed through a fluid, the viscous foree F_opposing the motion, is found imentally to depend on the radius r, the velocity of the sphere v-and the viscosity n ofthe flid, SS so F=fnry) If the function is the product of power functions of n, x and », F= Kits, where K is the dimensionless constant, If the above relation is dimensionally correct, then [MLT3] = [MLT) (Ly (LT or (MET?) = (ML Po Equating the exponents of similar quantities x ox ty +2= 1 and—x—2=-2. Solving these for x,y, and 2, we get x 1 So it becomes F= Knyy On experimental grounds, K = 6m; sé F'= 6xnrv This is the famous Stoke's law. to move in a circular orbit uniformly, centripetal which infact depends upon mass (m), velocity (0) of the circle. Express centripetal force in ‘quantities. HEBD According to the provided information, Fe mr 3 F= bine 4) ‘where k is the dimensionless constant of proportionality and a,b, care the constant powers of m, vr respectively. ‘Now using the principle of homogeneity, comparing the power of like quantities on both the sides, we have a=1 Gi) bt+e=1 .filijand — b=2 Liv) Using (ii, (ii), and (iv), we have a= 1, b=2, and c= -1 Using these values in (i), F=k m'v?r = F=K ™® which i the desired relation HEBD According to the provided information, vec dtprg S vakapg Ai) Where k is constant of proportionality. Using principle of homogeneity WeLT| = (Ly MOT (MOLT = (ML 7 Comparing powers of like quantities on both the sides, we have, b=0 i) a-3bt+e=1 Lill) —2e=—1 ..fiv) Dimensions and Measurement 1.7 Using Gi, (i), and (iv), we have, a= 5,b=0.6=1/2 Using these values in (i), we have v= k= A! pg! = v= kJAg which is the required relation, = if the = a sm Planck's constant (h) are chosen as fundamental find the dimensions of mass in new system. RD Let m= GI or m= KeGr By substituting the dimension of each quantity in both the sides, (MILT) = KLE} (M2 2p (MEET [Mots Lene peta, By equating the power of M, L, and T in both the sides:—y + xt 3y+2¢=0,-¥-2y By solving above three equations, x = 1/2, y= 1/2, ard z= 1/2. mec! G8 hi? If velocity (V), force (F), and time (7) are chosen as funda- ‘mental quantities , express (a) mass and (b) energy in terms of VF, and T. HERB Let 1 = (Some number) M)"(FY(TY Equating dimensions of both the sides, we get MT = (1) LTP (MLIT?) [Ty SM Le 8 peice Geta=-1,6=1,e=1. — ‘M = (Some number) (V" F'T!) I= ey Similarly, we can also express energy in terms of V, F, and T. Let [£] = [Some number] (V} (FI (7 => (MET?) = (ML?) (LY [MLT?Y (TF = (META = [a 121808 Posh] > 1=b2=atb+c;-2=-a-W+e Get lye=l. = B= (Some number) VT or [E] = (VILE NET). ee [EBD The physical quantities such as angles, trigonometric ratios are dimensionless. We are given the relation: <— , dimensionless dimensionless As sin( fi) is dimensionless, hence, the dimensional formula for ‘should be same as dimensional formula for FIV" i wv) (WET) _ ap pips ry so, [a]= {EBRD The physical quantities such as logarithm, exponential, ‘numerical factors, etc., are dimensionless. It means toe, should be dimensionless. We are given the relation: Fv? dimensionless “dimensionless (FI?) (i) ('] =(A=er (ii) (ML? 77) “ry From (i) and (i) [a= alal=Mirt® LIMITATIONS OF DIMENSIONAL ANALYSIS ‘Although dimensional analysis is very useful, it cannot lead us 100 far due to the following reasons: ‘L. If dimensions are given, physical quantity may not be ‘unique as many physical quantities have same dimensions For example, if the dimensional formula of a physical quantity is (94L27"], it may be work or energy oF torque 2. Numerical constant having no dimensions [X] such as, (1/2), 1, 2x, ete., cannot be deduced by the methods of dimensions. 3, The method of dimensions cannot be used to derive relations other than the product of power funetions. For example, s=ut+(12)aP or sin ox cannot be derived by using this theory (try if you can). However, the dimensional correctness of these can be checked, 4. The method of dimensions cannot be applied to derive formula if in mechanics a physical quantity depends on ‘more than three physical quantities as then there will be {ess number (= 3) of equations than the unknowns (>3). However, still, we can check the correctness of the given equation dimensionally. For example, T= 2x i/mg! cannot be derived by theory of dimensions but its dimensional correctness can be checked. Even if a physical quantity depends on three physical quantities, out of which two have same dimensions, the formula cannot be derived by the theory of dimensions, ‘eg, formula for the frequency of a tuning fork f= (d/L?) y cannot be derived by the theory of dimensions but can be checked Peet ae ote Gal ee |. If.x = at + br, where x is the distance travelled by the body in kilometer while ris the time in seconds, then find the unit of b. AA force F is given by F = at + be, where tis time. What are the dimensions of a and 5? ‘The position of a particle at time ¢is given by the relation a= so —e-"), where vy is a constant and a> 0. Find the dimensions of v, and 0. Find the dimensions of physical quantity Xin the equation Force = : Density . The number of particles is given by n=-D™2—"! m= crossing a unit area perpendicular to X-axis in unit time, where m, and m, are the number of particles per unit volume for the value of x meant to x, and x,. Find the dimensions of D called diffusion constant. G4 The equation ofa wave is given by Y= Asin a(2-r) i where wis the angular velocity and vis the li it Find the dimension of k. ae ‘ where A and B i 4B are dimensional constants, then find the dimensional formula for 4B. 8, You may not know integration, but using dimensional analysis you can check on some results, Inthe integral a sin * (2-1) sinttesacot J Qax | Convert 1 MW power on a new system having basie ‘units of mass, length, and time as 10 kg, 1 dm, and 1 min, respectively. 10; Suppose we employ a system in which the unit of ‘mass equals 100 kg. the unit of length equals 1 km and the unit of time 100 s and call the unit of enerEy ‘eluo} (jouie written in reverse order), then what is the relation between eluoj and joule? Uk If] gems! =xNs, then what is the value of x? 12. With the usual notations, check if the following ‘equation S, =u+a(2s—1) is dimensionally correct ‘or not 13, Ifthe time period (7) of vibration of 2 liquid drop depends on surface tension (S), radius (1) of the drop, and density (p) of the liquid, then find the expression of 7. 14 If P represents radiation pressure, C represents the ~” speed of light, and Q represents radiation energy striking a unit area per second, then non-zero integers x,y, and z such that P* Q' Cis dimensionless, find the values of x, y; and = 15. If velocity (V), force (F), and energy (E) are taken as 7” fundamental units, then find the dinsensional formula for mass. kms? 2.(a]=[MLT™],[6)=[MLT“] 3.[a]= (7. = 7) 4. (MPT S(ET} 6.(7) 7. (MET 8n=0 9.2.16 10? unit 10. eluoj= 10* joule 11. 10°Ns 12. Dimensionally correct 14.2=1,y=-l,andz=1 15.(V2F'E] SIGNIFICANT FIGURES Significant figures inthe measured value of a physical quantity tell the number of digits in which we have confidence. Larger the din a measurement, greater isthe accuracy of the measurement, The reverse is also tue The following rules are observed in counting the number of significant figures in a given measured quanti: ‘All non-zero digits are significant. Example: 42.3 has three significant figures. Dimensions and Measurement 243.4 has four significant figures 24.123 has five significant figures rer ecomes a significant figure iit appears Peemesn °° non-zero digits 5.03 has three significant figures. 5,604 has four significant figures. 4,004 has four significant figures Leading eros or the zeros placed tothe lef ofthe umber are never significant ‘Example: 0.543 has three significant figures (0,045 has two significant figures. (0.006 has one significant figure ‘Trailing zeros or the zeros placed to the r Example ight of the number are significant. Example: 4.330 has four significant figures 433.00 has five significant figures. 343,000 has six significant figures. In exponential notation, the numerical portion of significant figures. ‘Example: 132 x 107 has three significant figures. 1.32 x 10* has three significant figures. In order to avoid confusion in counting the amber of significant figures, we usually express 2 measured quantity in scientific notation. By this, the number of significant figures are tlearly mentioned and do not change on changing the units. For illustrations, see the table below. ives the number 1500 mm 500 x 10? mm, = 1.500 m. = 1,500 x 10? em 500 x 10? km ROUNDING OFF ‘While rounding off measurements, we use the following rules by convention: If the digit to be dropped is less than 5, then the preceding digit is left unchanged 82 is rounded off to 7.8, again x = 3.94 is Example: x rounded off to 3.9 Irthe digit to be dropped is more than 5, then the preceding digit is raised by | Example: x = 6.87 is wounded off to 6.9, again v = 12.78 is rounded off to 12.8. Ifthe digit tobe dropped is 5 followed by digits other than 2er0, then the preceding digit is raised by 1. Example: x= 16.351 is rounded off to 16.4, again x= rounded off to 6.8. 9.758 is 1.20 Mechanics 1 If the digit to be dt preceding, iti et unchanged. Example: x = 3.250 becomes {5+ 12.650 becomes 12,6 on rounding of ithe digit tobe dropped is or followed by zeros hen the preceding digit is raised by 1, if it is odd “Example: «= 3.750 is rounded off to 3.8, again x= 16.150 18 rounded off t0 16.2 repped 5 or 5 fellowed by rok then the iseven. 3.2 on rounding off, again SIGNIFICANT FIGURES IN CALCULATION In most of the experiments, the observations of various in geurements are t0 be combined mathematically, .e., added, ‘Ripracted. multiplied, or divided as to achieve the final result. ‘Since all the observations in measurements do not have the same precision, it is natural that the final result cannot be more precise than the least previse measurement, The following two rules should be followed to obtain the proper number of significant figures in any calculation 1. The result of an addition or subtraction in the number having different precisions should be rounded off the same number of decimal places as are present in the ‘number having the least number of decimal places. The rule is illustrated by the following examples: a 333 ‘€ (bas only one decimal place) 31 +0313 367. < (answer should be rounded off one decimal place) Answer = 36.7 3.1421 0.241 + 0.09 & (has 2 decimal places) 34731 « (answer should be rounded off 2 decimal places) Answer = 3.47 62.831 © (has 3 decimal places) 24.5492 382818 « (answer should be rounded off 3 decimal places) Answer = 38.282 2. The answer to a multiplication or division is rounded off to the same number of significant figures as is possessed by the least precise term used in the calculation. The rule is illustrated by the following examples: a. 142.06 ~ 023 ‘© (two significant figures) 32.6738 & (answer should have two significant figures) Answer =33 b. 51,028 x_131 © (three sig. icant figures) + 6.8 7 ag 702112676 Answer ~ 0.21 ORDER OF MAGNITUDE Insci fn integer. The order an et represent the quant value of the quantity we ignore the li ‘more than five, [BOD Volume ~o" The side of the cube is m BBRBD Total mass = 2.3 + 0.00215 + 0.01239 = pers are expressed as: Number = or tat les between | and 10 and xi Mis a mite of quantity isthe power of 10 For determining this rower, the vied off. While rounding off, the last digits 5 or arait which is less than 5. If edn digits increased by 1-For example, wc iotms'= 10's" (ignoring <5) 10°” kg (as 9.1 > 5) entific notation, the numa has to be roun Speed oflight in vacuum Mass of electron = 9.1 x 107" KB = p Dw el Each side of a cube is of the cube up to approy (7.203) = 973.715 m" reasured upto 4 significant figures. Thus the final answer must be reported in 4 significant figures priate significant figures. Hence, volume of the cube, V = 373.7 m’. ‘The mass of a box is 2.3 kg. Two marbles of masses 2.15 g and 12.39 g are added to it. Find the total mass of the box to the correct number of significant figures. 31 kg ‘The total mass in appropriate significant figures will be 2.3 kg. ‘The mass and volume of a body are 4.237 g and 2.5 em’, respectively. Find the density of the material of the body in correct significant figures. EBD The answer to a multiplication or division is rounded off to the same number of significant figures as possessed by the Jeast precise term used in the calculation, The final result should retain as many significant figures as are there in the original ‘number with the least significant figures. In given question, density should be reported to two significant figures. Mass. 378 2Sem’ After rounding off the number, we get density = 1.7 a ind thickness of “1.005 ,005 m and 2.01 cm respective sheet to correct number enna HEED We are given, * Length (0) = 4.234 m * Breadth (6) = 1,005 m * Thickness (()=2.01 em Density = Volume 6948 gfem? metal sheet are 4.234 m, gh 4 significant figures) (A significant figures) =201 x 102 - Calculate volume ofthe ™ (3 significant figures) Therefore, volume of the sheet Ixbxt 4.234 » 1,005 « 0.0201 m? = 0,0855289 m” In multiplication or division, the number of significant figures in the product or quotient is same as the smallest number of significant figures in any of the factors Hence, the volume can contain 3 significant figures, therefore, rounding off, we get : Volume = 0.0885 m? 1. Thelength, breadth, and thickness ofablockaremeasured as 125.5 om, 5.0 em, and 0.32 em, respectively. Which ‘one of the measurement is most accurate? | _ 2 The length of rectangular sheet is 1.5.em and the breadth is 1.203 em. Find the area of the face of a rectangular sheet to the correct umber of significant figures Each side of a cube is measured to be 5.402 cm. Find ‘the total surface area and the volume of the cube in appropriate significant figures. ‘Taking into account the significant figures, what is the value of 9.99 m+ 0.0099 m? Find the value of the multiplication 3.124 x 4.576 correct to three significant figures. If the value of resistance is 10.845 Q and the value of | ccurrent is 3.23 A, the potential difference is 35.02935 V. Find its value in significant number. ‘With due regard to significant figures, add the following: (@) 953 and 0.324 (b) 953 and 0.625 (©) 953.0 and 0.324 (A) 953.0 and 0.374 & With due regard to significant figures, subtract (@) 035 from 7 (b) 0.65 from 7 | (©) 0.35 from 7.0 (@) 0.65 from 7.0 ‘A diamond weighs 3.71 g. It is put into a box weighing 1.4 kg. Find the total weight of the box and diamond to the correct number of significant figures. Calculate the area enclosed by a circle of diameter 1.12 m to the correct number of significant figures. Se @e Se @ ve LL. (a) Add 3.8 * 10 to 4.2 * 10% with due regard to significant figures. (b) Subtract 3.2 * 10% from 4,7 * 10 with due regard to, significant figures. (©) Subtract 1.5 * 10? from 4.8 * 10* with due regard to significant figures. 12. The length, breadth, and thickness of a metal sheet are 4.234 m, 1.005 m, and 2.01 em, respectively. Give the area and volume of the sheet to the correct number of, significant figures. 1. Thickness 2.1.8 em? 3. Total surface area = 175.1 cm?, volume ~ 157.6 cm? 4.10.00m 5.143 6.350V 7.(a) 953 (b) 954 (©) 953.3. (d) 953.4 8.(a)7 6 66 (64 9.14kg 10.0.9 m? 11. (@)4.6x 10% — ()4.7x104 (©) 4.6% 10 3.72 m?, Volume = 0.0855 m? Dimensions and Messuremen: 1.1% ~ ERRORS OF MEASUREMENT ‘The measuring process is essentially a process of comparison. Ingpite of our best efforts, the measured value of @ quantity is always somewhat different from its actual value, or true value. ‘This difference in the true value of a quantity is called error of ‘measurement. Absolute error: Absolute error in the measurement of a physical quantity is the magnitude of the difference between the true value and the measured value of the quantity. Let a physical quantity be measured n times. Let the measured value be a, a, dy .... dy, The arithmetic mean of these values is a, $a bot dy ‘Usually, a, is taken as the true value of the quantity, ifthe same is unknown otherwise. By definition, absolute errors in the measured values of the quantity are a, = a,—a, 4a,=a,-a, Aa,=a,-a, ‘The absolute errors may be positive in certain cases and negative in certain other cases. ‘Mean absolute error: It is the arithmetic mean of the magnitudes of absolute errors in all the measurements of the quantity. It is represented by Aa. Thus, [ay [+ | Aaa | +] Ady | Hence, te final sult of measurement may be writen s a=a,+Aa ta ‘his implies that any measurement of the quantity is Likely to lie between (a,, +Aa) and (a,, ~ a). Relative error or fractional error: The relative error or fractional error of measurement is defined as the ratio of mean, absolute error to the mean value of the quantity measured, ‘Thus, relative error or fractional error Mean absolute enor _ Za | Percentage error: When the relativelfectionel eror is xpreecd in pereeriage, wo call it percentage ero Thus, pereentage error= 8% 100% PROPAGATION OF ERRORS Error in sum of the quantities: Suppose v= a+b Let Aa = absolute error in measurement of a Ab = absolute error in measurement of b ‘Ax = absolute error in calculation of x, ie. sum of a and b: ‘The maximum absolute error in.x is Ax = (Aa + 4b). a+ Ad) 1699 Percentage error in the value of x= Error In difference of the quantities: Suppose v= a—b, 4.42 Mechanics! casuroment ofa i 2 absolute error in aie mor siterenan of ‘av bole wand a Pein ia (Aa + Al 4 vatvor v= AAP) 096 Percentage error in the cro in product of quantities: Suppone ax. i nent of a i= absolute error in measurement of * aah went of b ‘Ab = absolute error in measuret ‘Av © absolute error in ealculati avand b. of'x, ie, product of ,, Ae Ab “Te maximum factional error in. is “T= Af Percentage eror inthe value of «= (Percentage error in value of a) 4 (Percentage error in value of b) a Error in division of quantities: Suppose x= 5 Let Aa = absolute error in measurement of a Ab = absolute error in measurement of b Ay = absolute error in calculation of x, ic. division of a and b. ‘The maximum fractional error in.x is Percentage error in the value of: + (Percentage error in value of b) Error in quantity raised to some power: Suppose x =~ Let Aa = absolute error in measurement of a oo” Ab = absolute error in measurement of b ‘Ax = absolute error in calculation of x Percentage error in value of a) The maximum fractional error in xis “* = a(n nit) x a" 5 Percentage erorin the value ofx= (Percentage errorin value ofa) + (Percentage error in value of b) The quantity which have maximum power must be measured carefully because it's contribution to error is maximum. in an experiment gave the values 1.29, 36, 1.30, and 1.33. Calculate the mean elative error, and percentage error. BRBD Here, mean value of quantity measured, y= 229413341344 135413241364 1304133 y= ENF 134 $1354 1.324136 + 1.304133 x = 1.3275 = 1.33 (rounded off to two places of decimal). Absolute errors in measurement are: 5 Ax= 133-129-004; ay Ax = 1.33-134=-001; ar .33-1.32=4001; ax 33-130" +003; Ax ‘Mean absolute error, Ax 0) 4.0.01 + 0.02.4 0.01 + 0.03 +0.03 + 0.00 0.04 133 Percentage error = 40.015 x 100 = 1.5% ‘A physical parameter a can be determined by measurit parameters, cal sing the relation 2 = SARE. ‘maximum errors in the measurement of b, c,d, and ¢ are b,® 6,% d,%, and ¢,%, then find the maximum error in the ‘ofa determined by the experiment. BERD 0 = Hotta! So maximum error in ais given by (2x10) a ax = ae x 1004 Bx 100+ 7 «100 48,22 100 6 ¢ d e = (ab, + Be, + 1, + be, )% The relative density of material of a body is found by weighing it first in air and then in water. If the weight in air is (5.00 + 0.05) N and the weight in water is (4.00 + 0.05) N. Find the relative density along with the maximum permissible percentage error. RRB Weight in air = (5.00 + 0.05) N Weight in water = (4,00 + 0.05) N Loss of weight in water = (1.00 + 0.1) N Weight inair Weight loss in water Now relative density = ie, RD = 200#0.05 1.00+0.1 Now relative density with maximum permissible error 5.00 (0. On 5:00 (0.05 , 0.1. 1.00 ($3 7) 0£(1 + 10)% 0+ 11% ‘The period of oscillation of a: ‘simple pendulum in the is recorded as 2.63 s, 2.56 s, 2.42 8, 2.71 8, and 2.80 s, average absolute error, ‘ =2 Now |A7)| =2.63-2.62=0.01 AT, = 2.62 -2.56 = 0.06 1A7;| = 2.62-2.42 = 0.20 blnanon sn enue 1.18 \AT,| = 2.71 -2.63 [a7 =2.80-2.62=0.18 ‘Mean absolute error gr = MUL ATI + ATI 1714 AT 0.54 : = = =0.108=0.11 If there Positive error of S0% in the measurement of ve- Tosty ofa body, find the emor inthe measurement of kineic SD ‘The length and breadth of a field are measured as: I = (120 + 2) mand b = (100-4 5) m, respectively. What isthe area of the * x waframuesnmnoer tea] 4[4% 4 3% + 3% + 2%] = E12%. field? M_ Al db (2, 5 Now 44 At, AP .{ 4 a ee aa (ii5* wo) Here Am=0and "100 = 50% AA = 0.0667 XA AE . Now 4 =1-b= 120 x 100= 12000 m* FP x10 = 250 = 100% => AA = 0.0667 x 12000 = 800.4 m? iin 12 penchant 0 12000 + 800.4 = (1.2 + 0.08) x 10 m? The initial and final temperatures of water as recorded by an observer are (40.6 + 0.2)°C and (78.3 + 0.3)°C. Calculate the nulisrraniani 1388) ‘rise in temperature with proper error limits. In an experiment of simple pendulum, the time period mea- = (40.6 £02) sured was 50 s for 25 vibrations when the length ofthe simple BBB 6,=(406+0.2C, 6-783 +03"C. pendulum was taken 100 em. Ifthe least count of top watch is Rise in temperature, 0= 0,0, = 78.3 - 40.6=37.7°C 0.1 sand that of meter scale is 0.1 cm. Calculate the maximus A0=4A0+ 40) =40.2-+03)= 0.59 possible error in the measurement of value of g. Ifthe actzal Hence, a rae ‘Value of g atthe place of experiment is 9.7720 ms*, calculate rise in temperature = (37.7 + 0.5)°C pct oe (ERBion 23 [BIRD THe time period of simple pendlum's given by + The length and breadth of a rectangle are (5.7 + 0.1) em and Fi a 2 (G4 = 02) cm, respectively Calculate the area of rectangle an fF on =e with error limits. 8 ‘As and mare constants, the maximum permissible error in g is [BBB Here, /= (5.7 = 0.1) om, b= 3.4 +02) om given by 48-4 , 247 ‘Area, A= 1x b= 5.7 3.4= 19.38 om’ = 19.0 em’ (rounding off g t T totwo significant figures) Here AL = 0.1 em, L=1m=100em, AT= 0.1 s M4 (ML, ab a on, (24) w(). ee so)” 100 * (35 0. a+( 2) 22] x100 01408 *\57 "34 o = Mb yg = 1S x 938 = 21.48 Thevetore, 19.38 19.38 1.5 (rounding off to two significant figures); (19.0 # 1.5) om oss ‘Actual value g = 9.7720 m ap “ g ‘Aphysical quantity xis calculated from the relation x=" 7. Percentage error § x 100 Ifthe percentage error ina, b,c, and d are 2%, 1%, 3%, and _ 9.8596 - 9.7720 a x 100=0,8964% 49%, respectively, what is the percentage error in x? 9.7720 4.16 Mechanics 1 The distance covered by a body in time (5.0 + 0.6) s is (40.0 In Ohm’s law exy : is ‘periment, the potential drop across a resi 0.4) m, Calculate the speed of the body. Also determine the was measured as V5.0 V and the current was meacured | percentage error in the speed. i= 2,00 A. Find the maximum permissible errorin resistan (ERD Here s = 40.04 0.4 mand r= 5.04 0.6 s. Therefore, speed, an aD & ~ 52 400 i vat Os omst (#) : As v=, therefore, ae t v=50V > AV=0.1V i=200A 4 = Ai=0.01A Here As= 0.4m, s= 40.0 m, Ar= 0.68, ¢ dv_ 04 | 06 (Be) = (ot) 00% = 2.59 ta pot sp Ayn {0.01 +012) x8.0= 1.04 «(@) (4 2.00 |*100% = Value of R Hence, v = (8.0+ 1.04) ms“; percentage error From the observation, R -1. 8 5a. i (4x10) = 0.13 x 100 = 13% So, we can write R= (2.5 + 2.5%) 2 nLuarRATION 147) | ucoaFRATION 1-38) In Seark’s experinnt to find Young's mals he diameter In resonance tube experiment, the velocity of sound is given of wire is measured as D = 0.05 cm, the length of wire is L = by v = 2{(1, - 1). We found 1, = 25.0 cm and 1, = 75.0 cm. 125 em, and when a weight, m= 20 kg is the put, extension in If there is no error in frequency, what will be the maximum _wire was found to be 0,100 cm, Find the maximum permissible permissible error in the speed of sound? (Take f, = 325 Hz) error in Young’s modulus (¥). iB = 2 0-1, mg. 3) amg eae we in (; 2 Glade (av) =2f, (al, -dl,) ay (dV), = max of [2f(+ Al, + Al] = 2f, (Al, + Al.) 1,=25.0 em = 1, = 0.1 em (place value of last number) 1, = 75.0 em = 1, = 0.1 em (place value of last number) So the maximum permissible error in the speed of sound (@V),,, = 2(325Hz) (0.1 em +0.1 om) = 1.3 ms" Value of V=2f, (1,—,) =2(325Hz) (75.0 cm—25.0 em)=325 ms" So V =G25+1.3)ms" If the measured value of resistance R= 1.05 Q, wire diameter To find the value of g using simple pendulum, 7=2,00 s and! ‘d= 0.60 mm, and length / = 75.3 cm, then find the maximum (00 m were measured. Estimate maximum permissible error in g. Also find the value of g. oe ee ca (nd*/4) ‘permissible error in resistivity, P=" - fi 7 ant = 2n jE eon sa Ad Al wm (2) - Bot (28) ~ MeaAE (201280 one P dae eee 00) eee R = 1.052 AR=0.01 0 (least count) _ 41 4x10%1,00 | ‘ad= 0.60 mm — Ad = 0.01 mm (least count) ear | 1 = 75.3 - Al= 0.1 cm (least count) 012 0.01 mm O.1em dp) 2018. fem (2) os ool ( gaa) 753m So ge (loos02 ms? Which of the following length measurements is most et jeasurem ()2.0cm —(b) 200m —_(@) 2.000 em 2, Ina number without decimal, what is the significance of | zeros on the right of non-zero digits? | RA esearch worker takes 100 observations in an ‘experiment. If he repeats the same experiment by taking 500 observations, how is the probable error affected? 4, Which quantity in a given formula should be measured most accurately? Why? 5, Abody travels uniformly a distance of (13.8 + 0.2) m in atime (4.0 + 0,3) s, Find the velocity of the body within | error limits and the percentage error. 6. The error in the measurement of the radius ofa sphere is 1%. Find the error in the measurement of volume. | Given R, = 5.0 + 0.2 Q, and R, = 10.0 + 0.1 Q. What is ‘the total resistance in parallel with possible % error? ‘The value of resistance is 10.845 Q and the current is 3.23 A. On multiplying them, we get the potential difference = 35.02935 V. What is the value of potential difference in terms of significant figures? 9. The length of one rod is 2.53 cm and that of the other i 1.27 em. The least count of the measuring instrument is 0.01 cm. If the two rods are put together end to end, find | the combined length. “10, ‘The pressure on a square plate is measured by measuring ‘the force on the plate and the length of the sides of the plate by using the formula P = FIP. If the maximum errors in the measurement of force and length are 4% and 2%, respectively, then what is the maximum error in the ‘measurement of pressure? | | | 11. The density of a cube is measured by measuring its mass and the length of its sides. If the maximum errors jn the measurement of mass and length are 3% and 2%, respectively, then find the maximum error in the measurement of the density of cube. ‘The resistance R= Vii, where V= 100 +5 V andi=J0 +£0.2 A. What is the total error in R? 13, The length of a cylinder is measured with a meter rod | having least count 0.1 cm. Its diameter is measured with Vernier calipers having least count 0.01 em. Given that length is 5.0 em. and radius is 2 em. Find the percentage error in the calculated value of the volume 14. According to Joule’s law of heating, heat produced =P Rt, where J is current, R is resistance, and ¢ is time, If the errors in the measurement of J, R, and ¢ are 3%, 4%, and 6%, respectively, find error in the measurement of H. 15, A physical quantity ? is given by Cand D brings in the maximum 2. quantity among 4, B, percentage error in P? Dimensions and Measurement 1.15 LO 2. Not significant 3. Probable error reduces to 1/5 4, Quantity having higher powers 5.3.45 ms", 8.95% 6.3% 8.35.0 9. (3.80 + 0.02) em 10. 8% 1.9% — 12.7% 14.16% — 15, Quantity C -AWERNIER CALLIPERS It consists of a main scale graduated in em/mm over which an auxiliary scale (or Vernier scale) can slide along the length (igure below). The division of the Vernier scale being cither slightly longer and shorter than the divisions of the main scale, (pms In the common form, the divisions on the Vernier scale V are smaller in size than the smallest division on the main seale M, but in some special cases, the size ofthe Vernier division may be larger than the main scale division, Letn Vernier scale divisions (VSD) coincide with (n~ 1) main scale divisions (MSD) then nVSD = (n= 1) MSD or 1VSD= (=) MsD 7 1 MSD - 1 VSD = 1 MSD (2) uso = +sp ‘The difference between the values of one main scale division and ‘one Vernier scale division is known as Vernier constant (VC) or the Least count (LC). This is the smallest distance that ean be accurately measured with the Vernier scale. 1 n Thus, VC=LC= sp -1 ysp=( sp __ Smallest di Number of divisions on Verni jon on main {In the ordinary Vernier callipers, one MSD is | mm and | 10 VSDs coincide with nine MSD. a 9 1 VSD = —MSD = 0.9 SD = {MSD = 0.9 mm VC=1 MSD-1 VSD =1 mm-0.9 mm = 0.1 mm = 0.0lem READING A VERNIER CALLIPERS If we have to measure a length AB, the end A coincides with the zero of main scale. Suppose the end B lies between 1.0m and 1.1 ‘em on the main scale. Then, 1.0 em on ZERO ERROR AND ZERO CORRECTION If the zero of the vernier scale does not coincide with the zero of main scale when jaw B touches A and the straight edge of D | touches the straight edge of C, then the instrument has an error called zero error. Zero error is always algebraically subtracted from measured length Zero correction has a magnitude equal to zero error but its sign is opposite to that ofthe zero error. Zero correction is always, algebraically added to measured length. Zero error — Algebraically subtracted Zero correction > Algebraically added POSITIVE AND NEGATIVE ZERO ERROR If the zero of Vernier scale lies to the right of the zero of main scale, then the zero error is positive, and if it lies to the left of zero of the main scale, then the zero error is negative (when jaws A and B are in contact). Positive zero error = (N + x x VC) Here N is the main scale reading on the left of zero of Vernier scale and x is the Vernier scale division which coincides with any main scale division. Excess reading = 03 mm (zero errr) we pu’ an object, between the jaws gives 13.8 mm reading In which there is 0.3 mm excess rea which has tobe removed (subtracted), So, actual thickness = 13.8 - 0.3= 13,5 mm Excess reading (zero error) Observed reading So, we ean formulate it as (AICULATING POSITIVE ZERO ERROR nthe given figure, one division ofthe main scale is of 1 mm and the jaws of Vernier are touching each other. The fith division of semier scale coincides with a main scale division, [ 30 |i | snl Nene = 1 ase, L.C.= += 0.1mm fn this © 7! Hence, the zero error = +5 ¥ 0.1 =0.5 mm and, Zero correction = — 0.5 mm, Here this error is to be subtracted from the reading taken for measurement. {ALCULATING NEGATIVE ZERO ERROR ‘nthis situation, the jaws of vernier are touching each other. The zero of the vernier scale lies to the left of the zero of the main ‘gale. And the fourth division of vernier scale coincides with a rain scale division as shown in the figure. Also scale Nerier Scale 1 LC.=mm=0. 1 ipmm= 0.1mm Hence, the zero error =—4 x 0.1 and zero correction = 0.4 mm 0.4mm, In this case, the error is to be added in the reading taken for measurement. the following data: 10 MSDs = 1 cm, 10 VSDs zero of Vernier scale isto the right of the zero mark- Jn scale with 6 VSDs coinciding with MSDs and for length measurement is 4.3 em with 2 VSDs ‘main scale graduations. Estimate the length. mm BBD Least count of Vernier calipers 1 division of main scale 1 = —__l division of main S°E*___—_ == 0.1 mm jumber of divisions in Vernier scale 10 Dimensions and Measurement _ 1.17 Side of cube = 10 mm +1 «0.1 mm = 1.01 cm Mass 2.7368 Volume (1,01)° em (o correct number of significant figures) oO (CREW GAUGE (OR MICROMETE! SCREW) in general, Vert callpers can measure accurately up 0 0.01 em and for greater accuracy micrometer screw devices, €.g., screw ‘gauge spherometer, are used. These consist of accurately cut serev. Which can be moved in a closely fitting fixed nut by tuming it axially (See figure below). The instrument is provided wth two scales: Now, density = =2.66 gem” Circular thea) sale hu Linear (pte) scale (a) The main scale or pitch scale graduated along the exis oF the serew (b) The cap-scale or head scale H round the edge of the screw head CONSTANTS OF THE SCREW GAUGE Pitch: The translational motion of the screw is directly proportional tothe total rotation of the head. The pitch of the instrument is the distance between two consecutive threads of the screw whicl: is equal to the distance moved by the screw due to one complete rotation of the cap. If the screw advances by 5 mm for 10 rotatious of the cap, then the pitch = 5/10 = 0.5 mm. Least count: In this case also, the minimum (or least) measurement (orcount) of length is equal to one division on the head scale which is equal to the pitch divided by the total cap divisions. Thus, in the aforesaid illustration, ifthe total cap division is 100, then least count = 0.5 mm/100 = 0.005 mm. Object being measured Ratchet serew Rotating barrel Inner cylinder Mechanis In case of the figure giver 2 am marks. © Main scale has J Girular scale has SOivSIONS. fersSmplete rotation the sew advances PY > mn Main seale reading = Circular scale reading = 48 Pi Least count, LC.“ F5rno. of ‘on the circular scale Object thickness = Main scale reading " + Circular scale reading * L.C. = 3,5 mm + 48 * 0.01 =3.98 mm y mm 50 =i = 0.01 mm ZERO ERROR AND ZERO CORRECTION Ina perfect instrament, the zero of the main scale coincides with ‘the line of graduation along the screw axis with no zero error, otherwise the instrument is said to have zero error which is equal to the cap reading with the gap closed. This error is positive when zero line or the reference line of the cap lies above the line of ‘graduation and vice versa. The corresponding corrections will be just opposite. Zero correction is the invert of zero error: Zero correction = ~ (zero error) ‘Actual reading ~ Observed reading ~ Zero error = Observed reading + Zero correction Let us take an instrument in which in a complete rotation, the spindle of the instrument advances by 1 mm, There are 100 divisions on circular scale. Fig.) Pitch _ mm Total no. of divisions 100 on the circular scale In figure (a), the instrument has positive zero error. Excess reading (Zero error) = 7 % LC. =7 * 0.01 = 0.07 mm This error has to removed (subtracted). In figure (b), the reading of instrument = Main scale reading + Circular scale reading x L.C =3mm +7 0.01 =3.07 mm In this reading, there is 0.07 mm excess reading which has to be subtracted. Least count, L.C. Hence, actual reading = BBD Least count Observed reading ~ Excess reading .07 - 0.07 = 3.00 mm SAGO RI dimensions and a pitch of the diameter ofa wire of length 1 mm and the 47th circular le, Find the diameter of the wire . ‘A screw gauge ha\ 1 mms used to measure t “The main scale reading is ‘coincides with the main sca “Pith No. of divisions on circular scale Wire diameter “= Main scale reading + Circular scale reading * L.C. mm +47 x LC = | mm +47 = 0.01 = 1.47 mm Ina complete rotation, the spindle of a screw gauge advances by 0.5 mm. There are 50 divisions on circular scale. The main scale has 0.5 mm marks (is graduated to 0.5 mee least count. = 0.5 mm) Read the screw gauge shown below ‘in fig (a) and (b): —] ‘Object thickness =45mm +39(2500) = 439mm ‘© Ameasurement of a physical quantity is said to be accurate ifthe systematic error in its measurement is relatively very ow. On the other hand, the measurement of a physical quantity is said to be precise ifthe random error is small ‘© Errors are always additive in nature © For greater accuracy, the quantity with higher power should have least error. | ‘Absolute error is not dimensionless quantity Relative error is a dimensionless quantity. ‘Value of 1 part on main scale (s) ‘Number of parts on vemnier scale (7) ‘© Least count of Vernier calipers | © Least Count = a ie of I part a 3 (ee of I part i main scale (s) Vernier scale (v) Least count of vernier calliper = 1 MSD ~ 1 VSD where MSD = Main scale division and VSD = Vernier scale division «+ Least count of serew guaze | Pitch (p) = Number of parts on circular scale (n) 's Smaller the least count, higher is the accuracy of ‘measurement. | «+ Larger the number of significant figures after the decimal in a measurement, higher is the accuracy of 7 's Significant figures do not change if we measure a physical quantity in different unit | 1s When we add or subtract two measured quantities, the absolute error in the final result is equal to the sum of the absolute errors in the measured quantities. “s When we multiply or divide two measured quantities, the relative error in the final result is equal to the sum ‘of the relative errors in the measured quantities. | i Solved Examples B.2 2 the dimension formula for [a] and [f] (here 1 = time, f= force, v= velocity, x = distance). BB Siee [FP] = ME | souase bearers 16 =MUTS x il B-MUT? and [» -4| will also have dimension M'L7". ¥ 18) _spprs;qj=MET! faa) “MET [al MLT ‘of mass m hung at one end of the spring executes simple ‘motion. The force constant ofa spring is k while its lof vibration is 7. rove by dimensional method that the T= 2smlkis incorrect. Derive the correct equation, that they are related by a power law. WB ve given equation is 7 2m Taking the dimensions of both sides, we have ty. ter Mir? Asthe dimensions of two sides are not equal, hence the equation ‘SSincorrect. Let the correct relation be T'= Ct’ k*, where C is constant. Equating the dimensions of both sides, we get ()= Dy (wry MALT) = [Mo* LT] Lae Dimensions and Measurement 1.19 ind T on both sides, we get Comparing the powers of ML a+ b=Oand —2b = 1 and a= ‘Therefore. wre?ac[) neniter ac} ‘This is the correct equation. “The radius ofthe earth is 6.37 x 10* m and its mass is 5.915 x 10" kg, Find the earth's average density t0 significant figures. [BAUD Given mass of the earth (M) and radius of the earth (R) Hence, volume of the earth (V) oben FxG.nynts 37108) m? M 597510 VB an xi6s7x10%) ‘Average density (D) = M2 pales 4 c(3.142)x(63710° 005517108 kgm” 2 52x10? kgm"? _ (to three significant figures) “The density is accurate only up to three significant figures which jis the accuracy of the least accurate factor, namely, the radius of the earth. ‘thas been observed that Velocity of ripple waves produced in water depends upon their wavelength (A), density of water (p). and surface tension (7). Prove that v*<« T/Ap. (BAB According to the problem, yee kp dep: where k is a dimensionless constant. LT = L(ML) (MTP MOLT! = Me Ls Using the principle of homogeneity, we get b+c=0,a~3b =1,-2c=-1 Solving these equations, we get pee eae 2° 27° 2 So, the relation becomes v= kA?p'?7¥? =p y? o a= r Ap a Senne Ga determining the value of acceleration gravit using a le lowing obtenaions vee rece Length of the string (J) = 98,0 om Diameter of the bob (d) = 2,56 cm Time for 10 oscillations (7) = 20.0 s te Calculate the value of g with maximum permissible absolute error and the percentage relative error, [BIRD Time period fora simple pendutum is 7 = 2, : (i) @ where [nis the effective length ofthe pendulum equal 0 ( + 4) and time period equals T ze = 2.005 aren) From (), we get ¢ == et To calculate actual value of g ad ae [iss a+r) 7 AF *OS+1.28) _ 980 cms? =9.80 ms" (2.007 Error in the value of g: 3.2.4 22) anar..2( 22) glee T ltr T Further, since errors can never exceed the least count of the ~ ‘measuring instrument. So, A/= 0.1 em and Ar= 0.01 em, = 0.1+0.01 (2) 041.28) **(200) = =0.0011 +0.01= 00111 Thus, percentage error 92 x 100% =1.1% g and absolute error = Ag = g(0.011) = 0.11 ms* So, g= (9.80 ms* + 1.1%) =(9.80+ 0.11) ms* ‘The length and breadth of a rectangular sheet are 16.2 cm and 10.1 cm, respectively. Find the area of the sheet in appropriate significant figures and error. HEBBD According to the problem, length, /= (16.2 + 0.1) em Breadth, b= (10.1 +0.1)em ‘Area, A= 17 b= (16.2 em) (10.1 em) = 163.62 em? As per the rule, arca will have only three significant figures and error will have only one significant figure. Rounding off, we get, area 4 = 164 em? If AA is error in an area, then relative error is calculated as AA/A. Then Basra anes _Glom | Otem 1.014162 2.63 162em Wem 1627101 163.62 263 > 263 Ma Ax 28 em? = 63.62 2 9.63 om? - 163.62" 163.62 " ‘AA = 3 em? (By rounding off to one significant figure) Area, A= A+ AA = (16443) cm? iti 02 m, and B You measure two quantities as A = 1.0 m + 02 m, 2,0 m +0.2 m. Find the correct value for J/AB in appropri significant figures and error. [BRD According to the problem, A=1.0m40.2 m, B=2.0m-+0.2 m Let z = VAB = y(1.0)(2.0) = 1.414 m Rounding off to two significant digits, Z = 1.4m AZ Lad, Lab 1(02m +1(22)-o15 | Z 24°28 21m) 2\ 2m => AZ =Z(0.15) = 1.4 m(0.15) = 0.212 m AS Rounding off to one significant digit, AZ = 0.2 m The correct value for AB = 1.4 + 0.2m. The tithe for 20 oscillations of a pendulum is measured as 1, = 39.6 8; ¢,= 39.9 sand f, = 39.5 s. What is the precision in the measurements? What is the accuracy of the measurement? [RGB According to the problem, the time for 20 oscillations of pendulum, ¢, = 39.65, f, = 39.98 and , = 39.5s, (a) It is quite obvious from these observations that the least count of the watch is 0.1 s. As measurements have only one, decimal place, precision in the measurement = Least count of the measuring instrument = 0.1 s Precision in 20 oscillations = 0.1 s Ol Therefore, precision in 1 oscillation = 20 (0) Mean value of time for 20 oscillations is given by tht 39.6 +39.9+395 3 3 ‘Mean time period of the second pendulum = = 39.66 s . 3866 ag Rounding off the time period of second pendulum = 2 s Measured time period of the second pendulum = 0.005 = 1.995 s Accuracy of measurement is the maximum observed error and is given by = 1.995 ~ 1.980 = 0.015 s A physical quantity X is related to four measurable quantities 4, b, cand d as follows X'= a bY 2, The percentage error in the measurement of a, 6, ¢ and d are 1%, 2%, 3% and 4%, respectively. What isthe percentage error in quantity X? Ifthe value of X calculated on the basis of the above relation is 2.763, ‘o what valuo should you round off the result? | MEBD Percentage error in quantity xis given by x 100. ; x ‘According to the problem, physical quantity is X= a'B'e!"d2 percentage error in d Maximum percentage error in X's ie x0 =[2{ 22100} 3{ 210} ; @ > “(Some =+[20) +30)+50)+ 2¢a)p =2[2+6-4$8]=2039% Percentage error in quantity = 223.5% Mean absolute error in X= +40.235 = 20.24 (rounding-off upto ‘wo significant digits) On the basis of these values, value of X should have two significant digits only. Therefore, X= 2.8 ity of light c, Planck's constant h and gravitational G are taken as fundamental quantities, then express and time in terms of dimensions ofthese quantities. {BBD We have to apply principle of homogeneity to solve {his problem. Principle of homogeneity states that in a correct, ‘gation, the dimensions of each term added or subtracted must be same, ie., dimensions of LHS and RHS should be equal. ‘We know that dimensions of [i] = MET"); (c] = (LT (= (MPT) @ LamechG= mkehG x) where, kis a dimensionless constant of proportionality. ‘Substituting dimensions of each term in Eq. (i), we get [MLTY = LT" (MET YLT = peer] ‘Comparing powers of same terms on both sides, we get b-c=l ii) at+2b+3e=0 (ii) -a-b-2e div) ‘Adding Eqs. (ii), (il) and (iv), we get 2b = I= b= i Substituting value of b in Eq, (i), we got From Eq. (iv), @ -b-de Substituting values of band c, we get a = gat Putting values of a, b and c in Eq. (i), we get Dimensions and Measurement 1.24, m= ke H9G "2 = m= ky a Gi) Let Lech b= keh AW) where k is a dimensionless constant, Substituting dimensions of each term in Eq. (v), we get LT p (MLE " * (MIL'T 2p MeLemiep 024 ‘Substituting value of b in Eq (vi), we get ¢ From Eq, (viii), a=-b ~ 2e Substituting values of band c, we get 1)_.3 (i (3 Putting values of a, b and c in Eq. (v), we get hG eteevnge tof (ii) Let T= WG = T= keh (ix) where k is a dimensionless constant. ‘Substituting dimensions of each term in Eq, (ix), we get (err) = EL} x (MEPS =o ero] On comparing powers of same terms, we get b-c=0 at 2b+3 -a-b-2¢ ‘Adding Eqs. (x), (xi) and (xii), we get 2 0 1 Substituting value of b in Eq, (x), we get ¢ = b = From Eq, (xii), a=-b~2e-1 Substituting values of b and c, we get + Putting values of a, b and c in Eq, (ix), we get Taken? 7 = fhS body, square ofthe period of revolution Tis the cube ofthe radius of the orbit, Using cimensonal analy a dimensionless constant. Mechanics 1 BEBBD According to Kepler's third law, 7 a’, i.c., square of time period (7°) of a satellite revolving around a planet is proportional to cube of the radius of the orbit (a’) We have to apply Kepler's third law, Tepes Tor? Also, T depends on R and g. Let Tee ritgeR'=s T= kreReg? where kis a dimensionless constant of proportionality Writing the dimensions of various quantities on both the sides, we get (MT) = (LPL SSL = Neer] -2a=1 => From Eq. (ii), we get = Pcucnin at ltl _Jé- Aphysical quantity depends upon five factors, all of which have dimensions; then method of dimensional analysis (1) Can be applied (2) Cannot be applied (3) Depends upon factors involved (4) Both (1) and (3) 2. A student when discussing the properties of a medium (except vacuum) writes Velocity of light in vacuum = Velocity of light in medium This formula is (1) Dimensionally correct (2) Dimensionally incorrect (3) Numerically incorrect, (4) Both (1) and (3) 3 Given that T stands for time period and / stands for the length of simple pendulum. If gis the acceleration due to gravity, then which of the following statements about the relation 7° = (ig) is correct? (1) Itis correct both dimensionally as well as numerically. (2) It is neither dimensionally correct nor numerically. ) Itis dimensionally correct but not numerically. (4) Itis numerically correct but not dimensionally. 4 Suppose refractive index pis given as p= A+ where A and B are constants and 2 is wavelength, then dimensions of B are same as that of (1) Wavelength (2) Volume ) Pressure (4) Area 8 The best method to reduce random error is (1) To change the instrument used for measurement (2) To take help of experienced observer (G) To repeat the experiment many times and to take the average results (4) None of the above { Of the following quantities, which one has the dimensions different from the remaining three? (1) Energy density (2) Force per unit area (3) Product of charge per unit volume and voltage (4) Angular momentum per unit mass LY The quantities A and B are related by the relation A/B =m, where m is the linear mass density and A is the force, the dimensions of B will be (1) Same as that of pressure (2) Same as that of work (3) That of momentum (4) Same as that of latent heat BC Which of the following quantities has its unit as newton- second? (1) Energy 3) Momentum (2) Torque (4) Angular momentum % 10. uM. 12. 14. 15, Which of the following is the most precise instrament for ‘measuring length? (1) Meter rod of least count 0.1 em (2) Vernier callipers of least count 0.01 em (3) Screw gauge of least count 0.001 em (4) Data is not sufficient to decide ‘The equation of the stationary wave is 2nct 2mx Aa 22 oo 2) Which of the following statements is wrong? (1) The unit of is same as that of A. (2) The unit of x is same as that of A. 3) The unit of 2nc/A is same as that of 27/2 (4) The unit of c/2 is same as that of x/A. 2 Given that: veal 22}or-9], where y and x are measured in the unit of length. Which of the following statements is true? (1) The unit of 2 is same as that of x and A. (2) The unit of A is same as that of x but may not be same as that of A. (3) The unit of c is same as that of 2702. (4) The unit of (ct ~ x) is same as that of 27/4. In the relation ® -resintor+e), the dimensional formula for a+ 9 is () MLT (2) MLD. @) MEP 4) MeL? ‘The length /, breadth b, and thickness ¢ of a block of wood ‘were measured with the help of a measuring scale, The results with permissible errors (in em) are 1= 15.124 0.01, b= 10.15 + 0.01, and t= 5.28 = 0.01 ‘The percentage error in volume up to proper significant figures is (1) 0.28% (2) 035% (3) 0.48% (4) 0.64%. A physical quantity x depends on quantities y and < as follows: x = Ay + Btan (Cz), where A, B, and C are constants. Which of the followings do not have the same dimensions? (1) xand B (3) y and BIA 2) Cand 2 (4) xand A The relative density of a material of a body is found by weighing it first in ait and then in water. If the weight of the body in air is 17, = 8.00 + 0.05 N and the weight in water is W, = 6.00 + 0.05 N, then the relative density ,= WAU, — W,) with the maximum permissible error is (1) 4.00 + 0.62% (2) 4.00 + 0.82% 3) 4.00 43.2% (4) 4.004 5.62% tions of a mass m suspended from oss e terms of time f and distance by « ea fn Ae jeoe De ‘Then the dimensions of A/B and F=Asin o i 2), [MoL-'T® oaert) (wert!) 2) (MLT*), (MOET?) @) ent7?), ALT A) (MOLT) [MoLsT") ' y=rsin (or — kx), the dimensions of 17, In the relation fare - a (eL'T") Q) —_ @ BELT] @ eer) ; 18. Given that Y= a sin a+ bt + cfcos cot. The unit of abe is ‘same as that of ay (2) it 3) oF (4) Git? 19. The frequency (n) of vibration of a string ‘is given as 1 [Z where T is tension and / is the length of "IV vibrating string, then the dimensional formula is @ perry @ BLT) @ fer] (4) (MET?) 20. If frequency F, velocity V, and density D are considered fundamental units, the dimensional formula for momentum will be Q) DVIF*! (4) DV4F 21. The potential energy of a particle varies with distance x as =A where and B are constants. The dimensional 4B formula for A * Bis (y Miver? @ miners (3) MEAT @ wert 22. If force F, acceleration a, and time T are taken as the fundamental physical quantities, the dimensions of length con this system of units are () ar (2) FAT (G3) FT (4) AT 23, The postion xofa particle at time tis given by x = Y0 (1 — ee"), where V, is constant and a > 0. The dimensions of V, and oare () M°LT™! and (2) M°LT® and T? (3) M°LT*' and LT? (4) M°LT and T 4. If-cand a stand for distance, then for what value of n is the given equation dimensionally correct? The equation is ao (22 ()-2 (41 5. The time dependence of a physical quantity P is given by P= Re", where Oris a constant and 1 is time. Then constant @is/has (1) Dimensionless (3) Dimensions of P (2) Dimensions of 7 (4) Dimensions of 7* 26. 2. 29. 30. 31. 32. The frequency fof vibra 1 spring of spring constant Cis a dimensionless cons 1 kis given by f= Cm'l’, where tant. The values of x and y are, respectively, » 1! O Fg Q) 2 11 at 5 ) 565 ‘A. student writes four different expressions for the displacement y in a periodic motion as a function of time ‘1.a.as amplitude, T'as time period. Which of the following] can be correct? IfC (the velocity of ight)g, (the acceleration due to gravity), | and P (the atmospheric pressure) are the fundamental quantities in MKS system, then the dimensions of length will be same as that of () Cig (2) CIP (3) PC 4) Clg | The relation tan 9= */rg gives the angle of banking of the cyclist going round the curve. Here v is the speed of the cyclist, ris the radius of he curve, and gis the acceleration due to gravity. Which of the following statements about the relation is true? (1) Itis both dimensionally as well as numerically correct. (2) It is neither dimensionally correct nor numerically correct. (3) Itis dimensionally correct but not numerically. (4) Itis numerically correct but not dimensionally. A physical quantity X’is represented by X'= (MILT). The ‘maximum percentage erors in the measurement of M, L, | and 7, respectively, are a%, 5% and c%. The maximum | Percentage error in the measurement of X will be | (1) (ax + by -c2)% (2) (av =by % (3) (ax+ by + e2)% (4) (ar ~by + c2)% The velocity of transverse wave in a string is v= /T/m where Tis the tension in the string and m is the mass per | unit length. If 7= 3.0 kgf, the mass of string is 2.5 g and the length of string is v= 1,C00 m, then the percentage error in the measurement of velocity is (os (2) 0.7 (3) 23 (4) 3.6 | Write the dimensions of a/b in the relation P -| bi where P is the pressure, «is the distance, and is the time. () Mer Q) MET? (3) ML°T? (4) MLT= ». Write the dimensions of a * b in the relation E=——— + where E is the energy, x is the displacement, and ¢ is the time. (y MET (@) MPT! @ MET? (4) mer? ifthe velocity of light C, the universal gravitational constant {G,and Planck's constant are chosen as fundamental units, the dimensions of mass inthis system are (y necrnG? @ mc @) hCG" (4) ACG ‘The effective length of a simple pendulum is the sum of the following three: length of string, radius of bob, and Jength of hook. Ina simple pendulum experiment, the length ofthe string, asmeasured by a meter scale, is 92.0 em. The radius of the bob combined with the length of the hook, as measured by a vernier calipers, is 2.15 cm. The effective length of the pendulum is (1) 94) om (2) 94.2.¢m @) 94.15 cm (4) 94.em ‘The moment of inertia ofabody rotating about given axisis 12.0kg:m* in the SI system. What is the value of the moment of inertia in a system of units in which the unit of length is em and the unit of mass is 10 e? (1) 2.410% (2) 6.0x10° @) 5.4x10° (4) 4.8x10° If the velocity (V), acceleration (A), and force (F) are taken as fundamental quantities instead of mass (0), length (L), and time (7), the dimensions of Young's modulus (¥) would be () Favs (2) Favs (@) FAV? (4) Fav? ‘The percentage errors in the measurement of mass and speed are 2% and 3%, respectively. How much will be the maximum error in the estimation of KE obtained by measuring mass and speed? (1) 5% @) 8% ‘An experiment measures quantities a, band c, and then X 1252 a “SF If the percentage errors in 2) 1% (4) 11% is calculated from 4 band c are +1%, 3%, and #2%, respectively, then the percentage error in X can be (1) £12.5% @) 4% ‘The resistance of a metal is given by R = VJ, where V is potential difference and / is the current. In a circuit, the potential difference across resistance is V=(8+0.5) V and ‘current in resistance, = (4 + 0.2) A. What is the value of resistance with its percentage error? () @ 45.6%) 2 Q (240.1%) 2 (3) @£35%) Q (4) 2 11.25%) @ (2) 47% (4) 44% a. 45. 47. 48. mension a Voxavect 42% eccand ger unit area val ter I adh vr, wher: the velocity, then the “The mass ofthe liquid lowing per ‘The mection of the tube i proportion Pia the pressure difference and v i 1 felation between x and y is (yey (rey (Ayyaot ay yree ; ‘A physical quantity x in calculated from x ~ all ‘ in measuring 7 When percentage ero i ere bh, and care 4, 2, and Ws, pereentage errors in measuring % respectively (1) % (2) % (3) 11% (4) 9.5% ‘The specific resistance p of a circular wie of sadist, F, Tesistce Rand length is given by p= 18. resi ips £0.02 cm, R= 30 £ | 2, and = 4.20 20005 0. ‘The percentage error in pis neatly ()% (2) % @) 13% (4) 20% Using mass (), length (L),time (7,and cleric ourers A) ia fundamental quantities, the dimensions of permitivity will be () (META (2) (MLT 7A) (3) (MLA T#A?] (4) (M277 4A) ‘Assuming thatthe mass m of the largest stone that can be ‘moved by a flowing river depends upon the velocity ¥ of the water, its density p, and the acceleration dive to gravity ‘g. Then m is directly proportional to ay (2y¥ 3) av ‘A spherical body of mass m and radius ris allowed to fal ina medium of viseosity 7. The time in which the velocity of the body increases from zero to 0.63 times the terminal velocity (v) is called time constant (7). Dimensionally. can be represented by re |67 Mm @ emir 6m e os (4) None of these oxy AA liquid drop of density p, radius r, and surface tension 6 oscillates with time period T. Which of the following ‘expressions for 7? is correct? 3 a = Q) ey ro oa (4) None of these Ahighly rigid cubical block 4 of small mass M and side L is fixed rigidly on the other cubical block of same dimensions and of modulus of rigidity 7 such that the lower face of A completely covers the upper face of B. The lower face of Bis rigidly held on a horizontal surface. A small force F is applied perpendicular to one of the side faces of A. After the force is withdrawn, block 4 executes small oscillations, the time period of which is given by () an {itn (2) 2M ©) tefl (4) 2nJMinL 26 9, 56. Mechanics I “The mass ofa body is 20.000 g and its volume is 10.00 em. Ifthe measured values are expressed to the correct significant figures, the maximum error in the value of density is (1) 0.001 gem? (2) 0.010 gem” (3) 0.100 gem? (4) None of these ‘The length of a strip measured with « meter rod is 10.0 om. Its width measured with a vernier callipers is 1,00 em. The Teast count of the meter rod is 0.1 em and that of vernier ccallipers is © 01 em, What will be the error in its area? (1) 40.01 em? 2) 40.Lem? 40.11 om? (4) £0.2 em? The relative density of a material is found by weighing the body first in air and then in water. Ifthe weight in ai is (10.0 £ 0.1) gf and the weight in water is (5.0 + 0.1) gf, then the maximum permissible percentage error in relative density is wo. @2 @3 as The dimensional formula for 2 physical quantity x is [M-ILT-2], The errors in measuring the quantities M, L, ang T. respectively. are 2%, 3%, and 4%, The maximum percentage of error that occurs in measuring the quantity xis ws 2) 10 @)14 (4) 19 The heat generated in a circuit is given by Q = PRE, where is current, ® is resistance, and fis time. If the percentage errors in measuring /.R.and are 2%, 1%, and 1%, respectively, ‘hen the maximum error in measuring heat will be ()% 2) 3% 3) 4% (4) 6% The internal and external diameters of a hollow cylinder ‘are measured with the help of a Vernier callipers. Their vvalues are 4.23 + 0,01 em and 3.87 + 0.01 em, respectively. The thickness of the wall ofthe cylinder is (1) 036 = 0.02 em (2) 0.18 + 0.02 em (3) 0360.01 em (4) 0.1840.01 om To determine the Young's modulus of a wire, the formula Bee 4 section of the wire, AL. = change in length ofthe wire when stretched with a force F. The conversion factor to change it from COS to MKS system is at (2) 10 (3) 01 (4) 001 ‘An experiment shows that two perfectly neutral parallel metal plates separated by a small distance d attract each other via a very weak force, known as the Casimir force, The force per unit arca ofthe plates, #, depends only on the Planck constant f, on the speed of light c, and on d. Which of the following has the best chanee uf being correct for /"? where L = length, A = area of cross- he he 1 Fae yt FR a ; A @ Fete a) r= ¢ 57. 59. 61. 2. 63. racy of measurement and significant context of accul mn ‘ ilk Tesults of experiment, which of the figures in expressing following is/are correct (@) Out of the two measurements 50.14 em and 0.00025 ampere, the fist one has greater accuracy. (0) Ione travels 478 km by rail and 397 m by road, the total distance travelled is 478 km. (1) Only (a) is correct (2) Only (b) is correct (3) Both are correct (4) None of them is correct The relative density of material of a body is found by | weighing it first in air and then in water. If the weight in | air is (5.00 + 0.05) Newton and weight in water is (4.00 ‘1'0.05) Newton. Then the relative density along with the maximum permissible percentage error is (1) 5.04 11% (2) 5.041% (3) 5.0+ 6% (4) 1.25 45% 1 ‘The focal length fof a mirror is given by where wand v represent object and image distances respectively. af au ® ra u v of Q) = f 8) of A(u+v) i Tay | of Au) Av | (4) ® fou vy uty uty | ‘The external and internal diameters of a hollow cylinder are measured to be (4.23 + 0.01) om and (3.89+ 0.01) cm. The thickness of the wall of the cylinder is (1) (0.34 £0.02) em (2) (0.17 £0.02) em (3) (0.17 £0.01) em (4) (0.344 0.01) em ‘A gas bubble from an expression under water, oscillates with a period 7' proportional to p'a*E*, where p is the static pressure, d is the density of water and £ is the total energy of explosion. Find the value of i Nes Ms L @) 4 In the relation Pp the distance, X is the Boltzmann constant and ce is the temperature. The dimensional formula of B will be: () [M2 7°) Q) (M'eTY @ IM ry @) WR Ty ‘A bus wavels distance x, when accelerates from rest nite a, for some time and after that travels a *, When decelerates at a constant rate a, to come to aa? rest. A student established a relation x, +, = 12" _ 2(a, +a) Choose the correct option(s). (1) The relation is dimensionally correct {2) The relation is dimensionally incorrect r (3) The relation may be dimensionally correct (4) None of the above |, Abody travels uniformly a distance of ($+ AS) “ (£41). What may be the condition so that panies ero limits move with velocity (Se As 5) ‘ ar Q) =41 9. “4 = or (asyee oa doe @abo ad oa ob I f at _ af? 3, Wunitoftmass becomes 2 times, the unit of length hecomes, 4 times and the unit of time becomes 4 times in the unit of Planck's constant. Due to this unit of Planck’s constant ‘becomes times. Find the value of n 1. If equation value of x, 4. Astone is lying at rest in a river. The minimum mass of stone, m= kpv"g? is needed for remaining at rest. constant having no unit, g = acceleration due to gravity v= river flow velocity, p = density of water. Find the value of x. 4. Ifthe unit of velocity is run, the unit of time is second and unit of force is strength in a hypothetical system of unit. In this system of unit, the unit of mass is (strength (second) (run). Find the value of yx If force F, velocity v and time T'are taken as fundamental units. Find the dimension of force in the dimensional formula of pressure. coi on "+ tan” (ax). Find the value of n. +. If volume is written as, V=Kg'e’A’, Here, K is dimensionless constant and g, c, h are gravitational constant, speed of light and Planck’s constant, respectively. Find the value of x/z & Acceleration due to gravity on the surface of the earth is ge a. ‘The gravitational constant G is exactly known. But percentage error in measurement of the mass of earth M and radius of the earth R are 1% and 2%. Respectively. The ‘maximum percentage error is measurement of acceleration due to gravity on the surface of the earth in n%. Find the value of 9. During measurement of kinetic energy 7, the percentage ‘error in measurement of mass of particle and momentum of particle are 2% and 3%., respectively. The percentage error in measurement of kinetic energy is n%. Find the value of n. 10, Ify=2.21 » 0.3, then find the number of significant digits in the value of y. M1, If x=0.72+0.8+3.87-1.089, then find number of significant digits in the value of x. 13. 15. 16. v7. 18, 19. 20. Dimensions an “The length of a wire is 2.17 cm and radius is 0.46 em. Find number of significant digits in the value of volume of wire. ‘The density of material in CGS system of units is 4 glec. In a system of units in which unit of length is 2 m and unit of mass is 16 g, find the numerical value of censity of material In a new system of units mass, acceleration and frequency are taken as fundamental units. If unit of mass is 100 g, unit of acceleration is 2 mis? and unit of frequency is 4 sec in the new system of units, then find the value of 0.1 Jin this system. Ina new system of units, the net force applied on a block of mass 10 kg moving with acceleration 10 m/s is given 1s 100 unit of force. When the same block is moving at 1 speed of 20 mis, its kinetic energy become 20 unit of energy. A liquid has a surface tension of 10 SI units. What js the magnitude of its surface tension in new system? (Dimensional formula of surface tension is ML'T") In an ancient civilization, the unit of length was half of meter, the unit of mass was double the kilogram and the ‘unit of time was half of second. Find the ratio of their unit of energy and Joule. Ina new system of unit, 1 unit of mass is 10 kg, 1 unit of length is 10 m and 1 unit of time is 1 s. If | unit of power in this unit is equal to x Wat, find x. ‘An experiment was conducted to measure the velocity of water splash when a car passes over a water layer on the road. It is found that velocity depends on the weight of car. power delivered by its engine arid thickness of water layer. ‘The velocity of splash was found to be 3 mis when a car ‘weighing 1000 N passes over a layer of thickness 3 mm and its engine is supplying a power of 800 KW. Find the velocity (in m/s) of splash when car weighing 500 N passes over a layer of thickness 4mm and its engine is supplying a power of just 400 kW. In a new system of units, the unit of mass is 100 g, unit of length is 4 m and unit of time is 2 s, Find the numerical value of 10 J in this system. The main scale of a Vernier calipers reads in millimeter and its Vernier is divided into 10 divisions which coincides with 9 divisions of the main scale. Find the reading for shown situation in mm, r a eT JEE Main JEE ADVANCED ‘Single Correct Answer Type Single Correct Answer Type 1 Ina exerinent ales ae rqud 19 be messed 1. A Vernier lies ha mn mks on he min cle using an instrument 29 divisions ofthe main scale exactly has 20 equal divisions on the verier scale, which may coincide with 30 divisions of the vernier scale. If the With 16 main scale divisions, For this Vernier callpes smallest division of the main scale is half-a-degree (=0.5°), lediat count is then the least count of the instrument is ayocran oe (0) one minute eed @) 0.1 mm 4) 0.2 mm Sone seer AIEEE 2009) (HT IEE an a eer towing eet rls obtained by measuring the 2+ The density of a solid ball is to be determined in an curent flowing in it and the voltage difference applied experiment in an experiment. The diameter of the ball | across it. Ifthe percentage errors in the measurements of ‘measured with a serew gauge, whose pitch is 0.5 mm and the current and the voltage difference are 3% each, then there ate 50 divisions on the circular scale. The reading on error in the value of resistance of the wire is the main scale is 2.5 mm and that on the circular scale (6% Q) zero 20 divisions. Ifthe measured mass of the ball has a relative @) 1% 4) 3% ervor of 2%, the relative percentage error in the density is (AIEEE 2012) (0.9% @) 24% 3. Let] denote the dimensional formula ofthe permittivity ) 3.1% (4) 42% of vacuum. If M = mass, L = length, T= time and (OT JEE 2011) | A= electric current, then: (IEEE TA tee) 8 etm ps (158) G)[eo]=(M"'PT' 4] @) (=(M"'LT A} by using Searle’s method, a wire of length L = 2 m and | (SEE Main 2013) 4. A student measures the time period of 100 oscillations of @ simple pendulum four times. The data set is 90 s, 91 s, 95s and 92 s. If the minimum division in the measuring diameter d= 0.5 mm is used. For a load M= 2.5 kg, an | extension /= 0.25 mm in the length of the wirs is observed. Quantities d and / are measured using a screw gauge and a Spey ig eieteerpar peed etn micrometer, respectively. They have the same pitch of 05 ea eta um, The number of divisions on their circular scale is 100 aaa aes The contributions tthe maximum probable sror ofthe Y GJEE Main 2016) measurement | A screw gauge witha pitch of 0.5 mm ads chen ne (1) due tothe errors in the measurements of ¢ and are the with 50 divisions is used to measure the thickness ofa thin same, | sheet of Aluminium. Before starting the peneerncet (2) dine to the error in the measurement of dis twice that | is found that when the two jaws of the serew gauge are due to the error in the measurement of brought in contact, the 45th division coincides with the (G) due to the error in the measurement of /is twice that ‘main scale line and thatthe zero ofthe main scale is barely due to the error in the measurement of d Visible. What isthe thickness ofthe sheet if the main scale (4) due to the error in the measurement of dis four times reading is 0.5 mm and the 25th division coincides the main that due to the error in the measurement of 7. sake le? 0 (IT JEE 2012) 175 mm 2) 0.80 mm : Ota an 4. The diameter of a eynder is eased sing a Verge ) (GRE Main 2016) callipers with no zero error. Its found thatthe zero ofthe | 7 Vernier scale lies between 5.10 em and 5.15 em of the main of a material in the shape of a cube is a ae canes scale The Vernier scale haa 50 divisions equieheste 243 iene me reeeecively 15 and Tete pace ore With one of the main seate divisions, The diameter of the in determining the density is cylinder is, (1) 6% 2) 25% () 5.112.em @) S.124em aa (4) 4.5% G) 5.136 em (8) 5.148 em (JEE Main 2018) (JEE Advanced 2013) Using the expression 2d sin @= A, one calculates the values of by measuring the corresponding angles 6 in the range ¢° to 90°. The wavelength A is exactly known and the error jn 6s constant for all values of @, As @ increases from 0°, (1) the absolute error in d remains constant. {Q) the absolute error in d increases. (3) the fractional error in d remains constant. (4) the fractional error in d decreases. GEE Advanced 2013) 4 There are two Vernier calipers both of which have 1 em divided into 10 equal divisions on the main scale. The Vernier scale of one of the calipers (C,) has 10 equal divisions that correspond to 9 main scale divisions. The ‘Vernier scale ofthe other caliper (C,) has 10 equal divisions that correspond to 11 main scale divisions. The readings of the two calipers are shown in the figure. The measured values (in em) by calipers C, and C,, respectively, are 2 3 4 ; i ‘ «thoi! wa | ( lulu: Lt | ny ea 10 (1) 2.87 and 2.87 (2) 2.87 and 2.86 (3) 2.87 and 2.83 (4) 2.85 and 2.82 (JEE Advanced 2016) 1. Apperson measures the depth of a well by measuring the time interval between dropping a stone and receiving the sound of impact with the bottom of the well. The error in his measurement of time is 97 = 0.01 second and he measures the depth of the well to be L = 20 meters. Take the acceleration due to gravity g= 10 ms” and the velocity of sound is 300 ms. Then the fractional error in the measurement, SL/L, is closest to (1) 0.2% (2) 5% (3) 3% (4) 1% (JEE Advanced 2017) Multiple Correct Answers Type 1. A student uses a simple pendulum of exactly 1 m length to determine g, the acceleration due to gravity. He uses a stop watch with the least count of 1 s for this and records 40 s for 20 oscillations. For this observation, which of the following statement(s) is(are) true? (1) Error A7 in measuring 7, the time period, is 0.05 s (2) Error AT in measuring 7, the time period, is 1 s @) Percentage error in the determination of g is 5% (4) Percentage error in the determination of g is 2.5% (UT JEE 2010) pimensions and Measurement 1.33 ac 1d gravitational jank’s constant h, speed of light ¢ an 2 Phamant are used to from a unit of length L and aunt of ass M. Then the correct options(s) is (are) () M«ve Q) Mee @) Levi @) Lave (SEE Advanced 2015) slectric current J, 3. In terms of potential difference v, el permittivity €, permeability 4, and speed of light the dimensionally correct ‘equations(s) is (are) (1) pol? <2”? (2) Hol = Hol” (3) 1-60 (4) jel =e (JEE Advanced 2015) 4. A length-scale (0) depends on the permittivity (€) of @ dielectric material, Boltzmann constant &,, the absolute temperature T, the number per unit volume (n) of certain charged particles, and the charge (q) carried by each of the particles, Which of the following expressions(s) for / is(are) dimensionally correct? P ek,T =| 0) I= eur) one) on[tt l_@#.) =i— I= @) l= [= =) @ (=) (JEE Advanced 2016) 5. In an experiment to determine the acceleration due to gravity g, the formula used for the time period of a period of a periodic motion is T = 2m, Maar) ‘The values of V 58 Rand r are measured to be (60 + 1) mm and (10+ 1) mm respectively. In five successive measurements, the time period is found to be 0.52 s, 0.56 s, 0.57 s, 0.54 and 0.59 s. The least count of the watch used for the measurement of time period is 0.01 s. Which of the following statement(s) is (are) true? (1) The error in the measurement of ris 10% (2) The error in the measurement of Tis 3.57% (3) The error in the measurement of Tis 2% (4) The error in the determined value of g is 11% (IEE Advanced 2016) Linked Comprehension Type For Questions 1 and 2 In electromagnetic theory, the electric and magnetic phenomena are related to each other. Therefore, the dimensions of electric and magnetic quantities must also be related to each other. In the questions below, [Z] and [B] stand for dimensions of electric and magnetic fields respectively, while [€,] and [4,] stand for dimensions of the permittivity and permeability of free space, respectively. [L] and [7] are dimensions of length and time, respectively. All the quantities are given in SI units (IEE Advanced 2018) 1, The relation between [E] and [B] is (1) (EI=(BN71 (2) (EI=(BET IT) (3) (E)= (ayy (4) (EI=(BIy't7y" 41.36 Mechanics 1 - 2. The relation between {e,] and [14] is : Numerical Value Type MbolleoNePTT’ |e) Les =fesItET"UrY 1, Tofind the distance d over which signal canbe seen clearly MMA T UTI tal Eea PTY in foggy conditions, a railways engineer uses dimensional analysis and assumes that the distance depends on the mass Matrix Match Type density p of the fog, intensity (power/area) S of the light 1. Match List with List I and select the correct answer using from the signal and its fequency The engineer nds enemas is proportional to $". The values of n is (IEE Advanced 2014), Boltzmann constant | 1. [ML!7 4-_Coefficientofviscosity | 2. [ML-"T™") 2. The energy of a system as a function of time ¢ is given as E= A? expat), wher a= 0.2 '. The measurement of 4 1__Planel constant 3 MLK] has an enor of 1.25%, Ifthe error inthe measurement of| cade conductivity Bo is 1.50%, the percentage error in the value of E() at 255 i=5sis w 3 1 2 4 GEE Advanced 2015) @3 2 1 4 @ 42 1 3 4 4102 3 GEE Advanced 2013) ee xencises sale Corect Answer Type aw a 4 24) 9.3) 2) Ww) me 1.0) 2) 4.0) ra) 29.41) 20) 0) 0) 3.0) 2) 40) 47.1) 49. (4) 52. (4) 54. (2) 57. 3) 59. (4) 62. (1) 64. (1) tapi Corect Answers Type L024) (2).3).4) 3..2).3) 4. (2),(4) 5. (1),2)4) 6. (1),2),G) 7. (3) 8.(1).G) 9. (1).2).3) 10. (1),(2) oh (1).2) 12. (1),2),G),(4) B. (1).C 14, (1).(2),3),(4) 15. (1),C 16. (2),(3),(4) 17. (1)43) 18. (2),(4) 19. (2).3) 20. (1).2).3) 2. (1)44) 22. (2),(4) 23. (1),2) 24. (1),(2).3)(4) Linked Comprehension Type Lil) 2. (3) 3.3) 4.(4) 6.14) 7. 3) 8B. (4) 9. (2) UL. (2) 12. (2) 13. (3) 14. (2) 16. (1) 17. (2) 18. (1) 19. (2) oimandan and Nesurenert_ 135 Answers Key : Matrix Match Type Live; ii adj ili > aivob Liveii aii divod 7 Rind acl bivred Hay A ioaiisaiiodgivoc 4 7 8. (2) 20. (4) 5.(1) 6. (3) ) 25 2) america Value Type 35. (2) 1. (0) 2. (8) 3. (6) 4.(1) 5. (1) 40. (4) 6. (1) 7.1) 8. (5) 9.(7) 10. (1) 45. (4) ‘AL. (2) 12. (2) 13. (2) 14. (4) 15. (1000) 50. (4) 16. (0.50) 17. (1000) 18. (3) 19. (25) 20. (6.9) 55. (3) 60. @) ARCHIVES JE Main Single Correct Answer Type 1.1) 2. G) 3.) 4.(1) 5. (2) 6. (4) Jt Advanced ‘Single Correct Answer Type 1.4) 2. 3) 3.) 4.(2) 5. (4) 6. (3) 7. (4) Multiple Correct Answers Type 1.(),G) 2. (),G)4) 3.(),G) 4.2.44) 5. (1),2)(4) Linked Comprehension Type 5. (1) 1.) 2. (4) 10. (2) 15. (1) Matrix Match Type L po4q>2%rohs33 Numerical Value Type 1.) 2. (4) INTRODUCTION ‘Mathematics is the supporting tool for physics. The elementary knowledge of basic maths is useful in problem-solving in physics The basic knowledge of elementary algebra, trigonometry, coordinate geometry, and calculus is must before going into the depth of physics. ELEMENTARY ALGEBRA COMMON FORMULAE 1, (a+ by + b+ 2ab 2. (a—bP =a" bab 3 (atb+cy + B+ 2+ 2ab + Ibe + 2ca 4 (a+ bYa-b)= a? 5. (a+b) a+b? + 3ab(a + b) @—b'—3ab(a—b) 7. (a+ bY (a—by’= dab 8 (a+ bP + (a— bP = 2a? +b’) POLYNOMIAL, LINEAR, AND QUADRATIC EQUATIONS Real Polynomial in», be real numbers and x be a real variable, then 1p a,x + a,x" +--+ +a,2" is called a real polynomial Degree or index of a polynomial: ‘The highest power appearing ina polynomial is called its degree. For example, s)=3° + 8x +3 is a polynomial of degree 3. ‘Students must note here that it is not necessary that the highest Power must be of a single variable only. For example, f(x, ») = 3ry +»? +2 is a polynomial of degree 3 because of the variable yin the term xy. We add the powers of the variables in a term to find the degree of a polynomial irrespective of the nature of variables. Thus, in the present case, x'y has power equal to 2+1=3. Hence, the degree of the given polynomial is 3. Linear Equations ‘Equations having terms of unit degree are called linear equations, ©g,x+y=2 or 2x + 3= 5, Such equations always represent a straight line on the graph. Quadratic Equations Equations of second degree are called quadratic equations, The general form of a quadratic equation is ax’ + bx + © = 0, where a0, Roots of a quadratic equation: ‘The solutions to a quadratic equation are ealled its roots. Roots are those values of a variable ‘such as x for which the given quadratic equation reduces to zero. AS a rule, a quadratic equation always has wo roots, which may or may not be equal “The roots of a quadratic equation are generally represented by and b, Let ax? + bx + ¢= 0 be a quadratic equation, ~b+ oP ~ Aa g _ ~b= li? ~ 4ac 1. Its roots are o oe aa Hence, its solution is given by x ‘Sum of ite roote is given by a+ 3. Product of its roots is given by of 4. Difference of its roots is given by a = BINOMIAL EXPRESSION AND THEOREM Analgebraic expression containing two terms is called a binomial cre fr caml(2+03. 02-9, (2+2}(22) se, bien Binomial Theorem for Positive Integral Index ‘The general form of a binomial expression is (x +a)", where nis any positive integer (called index) and x and a are real numbers. Binomial theorem states, (et ay arCe teat al HC FICE alt HC oat where "C, and n! = n(n — 1) = 2)..3 X21 r=) is the product of first natural numbers (¢.g,, 5! X4x3%2 ——————————— 2.2 Mechanics I jinomial coefficients: © THE expression al is read as “factorial.” So» Cae (n— (n= 2) 3X 2X1 VD! Hn ain 2) 3X 2X1 om nee... tent 2D 2x! © AC yPC AC, nC, are called bi Similarly, "C; | | | . | | | Binomial Theorem for Any Index Im is positive, negative. or fraction and that 1 << I. ie. Ties between 1 an the binomial theorem, mnt) 2 g MO=DO=2), 3 4. wo terms (asy stems MS * Note ‘© fnisa positive integer. then = Ifnisanegative integer or 8 the expansion will be infin ice, there will be no last term. «If ix = 1. then only the first two terms of the expansion are of higher order terms become ‘Sexy small and can be neglected. Thus, ia this case, the binomial following simplified forms when |x| <<1: significant because the values ‘expansion reduces to the: 2a 1 +n (+9) (-ay-1-m 0-3" Calculate (1001)'”, ms, = We can write 1001 es 1001 = 1000 (1 so that we have 13 1 ouw1)>= fio(s+tg)) f+ a 1000 1000 rea +0001= 11420001) 7 = 10,003333 Expand (1 +2)". 2 me o-e- 14 3px BSE B= 13-2) 3 g., 3 . 2__60 PPB 3x2 3x + 6x? ~ 102" + ‘The value of acceleration due to gravity (g) at height h above ee (R+hy the surface of earth is given by g/ = is any real number such \d +1, then according t© the expansion will have (n + 1) terms. fraction, then the number of terms in Ih <

You might also like